Vous êtes sur la page 1sur 87

1. Editorial: The Intercontinental Bank should reallocate the voting shares ofits.

Members in order to more effectively shape global economic policy. For example, China comprises about 15 percent of the world's gross domestic product but has only a 3 percent voting share, whereas Belgium, with less than 1 percent of the global economy, has a 2 percent share. Which of the following ls an assumption upon which the editorial's authors depend in suggesting a way to more effectively shape global economic policy? (A) The United States has a larger voting share of the Intercontinental Bank than does China or Belgium. (B) The specific allocation of voting shares factors into the Intercontinental Bank's effectiveness in shaping global economic policy. (C) Only voting shares that are precisely proportional to each countty's contrlbution to the global economy are appropriate for the Intercontinental Bank. (D) The Intercontinental Bank is necessary to the maintenance of a prosperous global economy. (E) As one of the fastest growing economies, China should have a larger voting share in the Intercontinental Bank. 2. For a local government to outlaw all strikes by its workers is a costly mistake, because all its labour disputes must then be settled by binding arbitration, without any negotiated public-sector labour settlements guiding the arbitrators. Strikes should be outlawed only for categories of public-sector workers for whose services no acceptable substitute exists. The statements above best support which of the following conclusions? (A) Where public-service workers are permitted to strike, contract negotiations with those workers are typically settled without a strike. (B) Where strikes by all categories of public-sector workers are outlawed, no acceptable substitutes for the services provided by any of those workers are available. (C) Binding arbitration tends to be more advantageous for public-service workers where it is the only available means of settling labor disputes with such workers. (D) Most categories of public-sector workers have no counterparts in the private sector. (E) A strike by workers in a local government is unlikely to be settled without help from an arbitrator. 3. Advertisement: According to a recent research study, daily use of Fresh Start, a new toothpaste, reduces the risk of developing dental cavities by over 20 percent. In addition, as a result of a new formula, the use of Fresh Start results in whiter, healthier-looking teeth. Clearly, Fresh Start not only gives your teeth a beautiful look but also provides the most reliable protection against dental cavities. Which of the following is an assumption in the argument above? (A) No other toothpaste provides more reliable protection against dental cavities. (B) Fresh Start's formula does a better job of whitening teeth than do competitors' formulas. . (C) People are just as interested in having beautiful teeth as they are in having healthy teeth. (0) Fresh Start also prevents other dental disorders, such as gingivitis.

(E) Reliable protection against dental cavities, combined with excellent aesthetic properties, is likely to make Fresh Start a popular toothpaste on the market. 4. Student Advisor: One of our exchange students faced multiple arguments with her parents over the course of the past year. Not surprisingly, her grade point average (GPA)over the same period showed a steep decline. This is just one example ofageneral troth: problematic family relationships can cause significant academic difficulties for our students. Which of the following is an assumption underlying the general truism claimed by the Student Advisor? (A) Last year, the exchange student reduced the amount of time spent on academic work, resulting in a lower GPA. (B) The decline in the GPA of the. Exchange student was not the reason for the student's arguments with her parents. (C) School GPA is an accurate measure of a student's intellectual ability. (D) If proper measures are not taken, the decline in the student's academic performance may become irreversible. (E) Fluctuations in academic performance are typical for many students, 5. Children born to older dads score lower on average on a range of tests, including concentration, memory, reasoning and reading, while the kids of older mothers score higher than those of younger mothers. Thats according to a University of Queensland study, which analysed data on more than 30,000 Australian children taken at 8 months and again at 4 years and 7 years, respectively. Which of the following conclusions is least supported by the above finding? A .Children born to older dads may have to pay a significant price- in intelligence. B. It is better for women to conceive at older ages. C. Older dads are financially well-established which ensures good nurturing of their children, leading to higher IQ. D. Younger women will give birth to dull children.

6. All acts have consequences. Given this fact, we may wish to play it safe by never doing anything. The speaker implies that A .we may prefer to live safely. B all acts have consequences. C consequentially is not safe. D. not doing anything is not an act 7.The maximum time an athlete is able to continue climbing till exhaustion sets in, may be the only determinant of his/her performance. A new European study, the objective of which is to help trainers and climbers design training programmes for this type of sport, shows this to be the case. Until now, performance indicators for climbing have been low body fat percentage and grip strength. Further, existing research was based on a comparison of amateur and expert climbers. Now, a new study carried out with 16 high level climbers reveals that the time an athlete takes to become exhausted is the only indicator of his/her performance. From the passage it can be concluded thatA the new approach of determining performance indicators is better than the previous one.

B the objective of this study is to help trainers and climbers design training programs with a higher difficulty level. C the level of performance can be judged by the time taken by an athlete till he gets too tired. D All of the above 8.Aristotle said that art represents general truths about human nature. Our city councilman is arguing in favour of the artistrya giant mural in front of a Jeep dealership, portraying a variety of four-wheel-drive vehicles. He cites Aristotles conception of art as his support. The passage above raises which of the following questions? A Can a city councilman understands Aristotle? B Which general truth about human nature does a four-wheel-drive mural not represents? C Could Aristotle have predicted a modern society filled with sophisticated machines? D What general truth about human nature does a mural of four-wheel-drive vehicles represent? 9. Consumers are not so easily manipulated as they are often painted. They may know what they want, and what they want may be greatly different from what other people believe they need. Which of the following statements, if true, most weakens the above argument? A Most people continue to buy the same brand of a product year after year. B Companies that advertise the most sell the most products. C Store shelves packed with a variety of different brands have the potential to confuse the consumer. D Most consumers know which brand they are going to buy before entering a store. 10. The department store owned by my competitor sells green necklaces that glow in the dark. Only those customers of mine wearing those necklaces must be giving business to the competition. The conclusion could best be strengthened by A deleting that glow in the dark. B. Changing sells to has sold. C. Changing the competition to my competitor. D. inserting only as the first word in sentence one. 11. All race-car lovers enjoy classical music. No backgammon players enjoy classical music. All those who enjoy classical music also enjoy fine wine. If each of the above statements is true, which of the following must also be true? A Everyone who plays backgammon enjoys fine wine. B No one who enjoys fine wine plays backgammon. C No backgammon players are race-car lovers. D No backgammon players enjoy fine wine. 12.It has been proven that the lie detector can be fooled. If one is truly unaware that one is lying, when in fact one is, then the lie detector is worthless. Without contradicting his or her own statements, the author might present which of the following arguments as a strong point in favour of the lie detector? A The methodology used by investigative critics of the lie detector is itself highly flawed. B Law-enforcement agencies have purchased too many detectors to abandon them now. C Circumstantial evidence might be more useful in a criminal case than is personal testimony. D The very threat of a lie-detector test has led a significant number of criminals to confess. 13.In parts of the world where the life spans are short, forty may be regarded as an advanced age. People who live longer are believed to possess special powers. These elders are sometimes treated with a deference based on fear rather than love. The final statement in the passage is based on which of the following assumptions?

A. Deference is normally accorded based on love. B. Few elders are treated with deference. C. People who live shorter lives have no special powers. D. People with special powers are not loved. 14. Samuel is obviously a bad fisherman. During the past season, in which he and the five members of his team spent four months on a boat together off Dutch Harbor, AK, he caught fewer fish than any of his teammates. Which of the following, if true, most weakens the argument above? A) Two seasons ago, Samuel fished on another boat off Dutch Harbor and caught more fish than any other member of that boat. B) Before becoming a fisherman, Samuel piloted a fishing boat whose members regularly caught record numbers of fish. C) While fishing this past season, Samuel fell sick for a week and did not catch any fish during this time. D) Unlike the other fishermen on his boat, at the order of the captain, Samuel fished this past season with experimental bait. E) Amongst the fishing community in Dutch Harbor, Samuel has a reputation for being an especially bad fisherman. 15. For a local government to outlaw all strikes by its workers is a costly mistake, because all its labor disputes must then be settled by binding arbitration, without any negotiated public-sector labor settlements guiding the arbitrators. Strikes should be outlawed only for categories of public-sector workers for whose services no acceptable substitute exists. The statements above best support which of the following conclusions? (A) Where public-service workers are permitted to strike, contract negotiations with those workers are typically settled without a strike. (B) Where strikes by all categories of public-sector workers are outlawed, no acceptable substitutes for the services provided by any of those workers are available. (C) Binding arbitration tends to be more advantageous for public-service workers where it is the only available means of settling labor disputes with such workers. (D) Most categories of public-sector workers have no counterparts in the private sector. (E) A strike by workers in a local government is unlikely to be settled without help from an arbitrator. 16.Which of the following best completes the passage below? People buy prestige when they buy a premium product. They want to be associated with something special. Mass-marketing techniques and price-reduction strategies should not be used because______ (A) affluent purchasers currently represent a shrinking portion of the population of all purchasers (B) continued sales depend directly on the maintenance of an aura of exclusivity (C) purchasers of premium products are concerned with the quality as well as with the price of the products (D) expansion of the market niche to include a broader spectrum of consumers will increase profits

(E) manufacturing a premium brand is not necessarily more costly than manufacturing a standard brand of the same product 17.A researcher discovered that people who have low levels of immune-system activity tend to score much lower on tests of mental health than do people with normal or high immune-system activity. The researcher concluded from this experiment that the immune system protects against mental illness as well as against physical disease. The researchers conclusion depends on which of the following assumptions? (A) High immune-system activity protects against mental illness better than normal immune-system activity does. (B) Mental illness is similar to physical disease in its effects on body system. (C) People with high immune-system activity cannot develop mental illness. (D) Mental illness does not cause peoples immune-system activity to decrease. (E) Psychological treatment of mental illness is not as effective as is medical treatment. 18. In January there was a large drop in the number of new houses sold, because interest rates for mortgages were falling and many consumers were waiting to see how low the rates would go. This large sales drop was accompanied by a sharp rise in the average price of new houses sold. Which of the following, if true, best explains the sharp rise in the average price of new houses? (A) Sales of higher-priced houses were unaffected by the sales drop because their purchasers have fewer constraints limiting the total amount they pay. (B) Labor agreements of builders with construction unions are not due to expire until the next January. (C) The prices of new houses have been rising slowly over the past three years because there is an increasing shortage of housing. (D) There was a greater amount of moderate-priced housing available for resale by owners during January than in the preceding three months. (E) Interest rates for home mortgages are expected to rise sharply later in the year if predictions of increased business activity in general prove to be accurate. 18. Years ago, some in the government's intelligence community feared the work of telecommunications researchers at then-emerging private security firms. The government experts concluded that these private firms posed the biggest risk to successful government espionage. As the private security firms began publicly releasing and advertising encryption algorithms and other security products, these government experts saw support for their conclusion when an encryption algorithm that government experts could not break began appearing in countless emails. Which of the following, if true, most weakens the conclusion of the government experts referred to above? A) Shortly before the government experts reached their conclusions, two private security companies each claimed to have developed "the world's strongest email encryption algorithm" B) The private security firms' decision to advertise their products and sell them publicly led to other members of the private sector and academia scrutinizing the encryption

algorithms. C) An open-source encryption algorithm, developed by an academic and freely available from popular websites, is recognized by numerous ex-government code breakers as the most unbreakable algorithm ever developed. D) An enemy government recently succeeded in placing a spy within the government espionage operations referred to above. E) To strengthen the reputation of the private security firms, employees of these firms publish information about the strength of their products and the benefits of using them. 19. One morning, George Petersen of Petersen's Garage watches as a 1995 Da Volo station wagon is towed onto his lot. Because he knows that nearly 90% of the 1995 Da Volo station wagons brought to his garage for work in the past were brought in because of malfunctioning power windows, he reasons that there is an almost 9 to 1 chance that the car he saw this morning has also been brought in to correct its faulty power windows. Which one of the following employs flawed reasoning most similar to that employed by George Petersen? (1) Mayor Lieberman was re-elected by a majority of almost 75%. Since Janine Davis voted in that mayoral election, the chances are almost 3 to 1 that she voted for Mayor Lieberman. (2) Each week nine out of 10 best-selling paperback books at The Reader's Nook are works of fiction. Since Nash's history of World War II was among the ten best-selling paperback books at The Reader's Nook this week, the chances are 9 to 1 that it is a work of fiction (3) 90% of those who attempt to get into Myrmidon Military Academy are turned down. Since the previous 10 candidates to the academy were not accepted, Vladimir's application will almost certainly be approved. (4) Only one out of 50 applications for bypassing zoning regulations and establishing a new business in the Gedford residential district is accepted. Since only 12 such applications were made last month, there is virtually no chance that any of them will be accepted. 20. Current farm policy is institutionalized penalization of consumers. It increases food prices for middle- and low-income families and costs the taxpayer billions of dollars a year. Which of the following statements, if true, would provide support for the author's claims above? I. Farm subsidies amount to roughly $20 billion a year in federal payouts and $12 billion more in higher food prices. II. According to a study by the Department of Agriculture, each $1 of benefits provided to farmers for ethanol production costs consumers and taxpayers $4. III. The average full-time farmers have an average net worth of over $300,000. (A) I only (B) II only (C) III only (D) I and II only (E) I, II, and III

21. Biologists attached a radio transmitter to one of a number of wolves that had been released earlier in the White River Wilderness Area as part of a relocation project. The biologists hoped to use this wolf to track the movements of the whole pack. Wolves usually range over a wide area in search of prey, and frequently follow the migrations of their prey animals. The biologists were surprised to find that this particular wolf never moved more than five miles away from the location in which it was first tagged. Which one of the following, if true, would by itself most help to explain the behaviour of the wolf tagged by the biologists? (1) The area in which the wolves were released was rocky and mountainous, in contrast to the flat, heavily- wooded area from which they were taken. (2) The wolf had been tagged and released by the biologists only three miles away from a sheep ranch that provided a large, stable population of prey animals. (3) The White River Wilderness Area had supported a population of wolves in past years, but they had been hunted to extinction. (4) Although the wolves in the White River Wilderness Area were under government protection, their numbers had been sharply reduced, within a few years of their release, by illegal hunting. 22. Commentator: The theory of trade retaliation states that countries closed out of any of another countries market should close some of their own markets to the other country in order to pressure the other country to reopen its markets. If every country acted according to this theory, no country would trade with any other The commentators argument relies on which of the following assumptions? (A) No country actually acts according to the theory of trade retaliation. (B) No country should block any of its markets to foreign trade. (C) Trade disputes should be settled by international tribunal. (D) For any two countries, at least one has some market closed to the other (E) Countries close their markets to foreigners to protect domestic producers. 23. Although computers can enhance peoples ability to communicate, computer games are a cause of underdeveloped communication skills in children. After-school hours spent playing computer games are hours not spent talking with people. Therefore, children who spend all their spare time playing these games have less experience in interpersonal communication than other children have. The argument depends on which of the following assumptions? (A) Passive activities such as watching television and listening to music do not hinder the development of communication skills in children. (B) Most children have other opportunities, in addition to after-school hours, in which they can choose whether to play computer games or to interact with other people. (C) Children who do not spend all of their after-school hours playing computer games spend at least some of that time talking with other people. (D) Formal instruction contributes little or nothing to childrens acquisition of communication skills (E) the mental skills developed through playing computer games do not contribute significantly to childrens intellectual development. 24. Monthly employee evaluations are an excellent tool for managers. With them, employees that are not productive can be removed from the company, and efficient workers rewarded, and all within the space of a month.

The argument above logically depends on which of the following assumptions? A.Workers do not in turn complete evaluations of the management of the company. B Unproductive employees often refuse to co-operate with the managers doing the evaluations, because they see these evaluations as a violation of their privacy. C Most studies indicate that employee evaluations have no bearing whatsoever on worker satisfaction. D Evaluations can identify with some accuracy whether an employee is productive or not. E Employee evaluations are a better source of information about employee grievances than are monthly staff meetings. 25. A CEO of a major company has noted a serious decline in worker productivity within the last 3 years. According to a report completed by a consultant, productivity dropped by 35% during that time. The CEO has therefore intiated a plan to boost productivity in which employees receive stock options in the company. Which of the following, if true, would make the CEO's plan logically sound? A. Studies indicate that owning part of something makes one work harder to make it successful. B Workers respond more to increases in salary than to the threat of being fired or demoted. C The drop in worker productivity is due to the management policies of the current CEO and the Board of Directors. D The financial situation of the company is actually more grave than the issue of employee productivity. E The outside consultant hired to study the productivity of the company is often very accurate in her work. 26. The socialistic ideals believe in the redistribution of wealth. If these socialstic ideals are not followed then there will be intolerable economic inequities. Once the economic inequities become intolerable the people who are marginalized resort to violence to coerce social reforms. So, a state should adopt socialist ideals to avoid any kind of violent attempts at social reforms. Which of the following is a premise that stands behind the argument? A. Any attempt at social reforms justifies resorting to violent means. B. There is no other ideology that believes in redistribution of wealth. C. Inequities make people resort to violent means. D. Democracy can be used to choose socialistic goals for a state. E. It's a state's responsibility to redistribute wealth. 27. A study of marital relationshipsn which one partner's sleeping and waking cycles differ from those of the other partner reveals that such couples share fewer activities with each other and have more violent arguments than do Couples in a relationship in which both partners follow the same sleeping and waking patternsThus, mismatched sleeping and waking cycles can seriously jeopardize a marriage

Which of the following, if true l most seriously weakens the argument above? (A) Married couples in which both spouses follow the same sleeping and waking patterns also occasionally have arguments that can jeopardize the couple's marriage. (B) The sleeping and waking cycles of individuals tend to vary from season to season (C) The individuals who have sleeping and waking cycles that differ significantly from those of their spouses tend to argue little with colleagues at work (D) People in unhappy marriages have been found to express hostility by adopting different sleeping and waking cycles from those of their spouses (E) According to a recent study, most people's sleeping and waking cycles can be controlled and modified easily 28. In the past most airline companies minimized aircraft weight to minimize fuel costsThe safest airline seats were heavy, and airlines equipped their planes with few of these seatsThis year the seat that has sold best to airlines has been the safest one-a clear indication that airlines are assigning a higher priority to safe seating than to minimizing fuel costs Which of the followingif truemost seriously weakens the argument above? (A) Last year's best-selling airline seat was not the safest airline seat on the market (B) No airline company has announced that it would be making safe seating a higher priority this year (C) The price of fuel was higher this year than it had been in most of the years when the safest airline seats sold poorly (D) Because of increases in the cost of materialsall airline seats were more expensive to manufacture this year than in any previous year (E) Because of technological innovationsthe safest airline seat on the market this year weighed less than most other airline seats on the market 29. Investing in real estate would be a profitable venture at this time. A survey in House magazine revealed that 85% of the magazines readers are planning to buy a second home over the next few years. A study of the real estate industry, however, revealed that the current supply of homes could only provide for 65% of that demand each year. i) Which of the following, if true, reveals a weakness in the evidence cited above? (A) Real estate is a highly labor-intensive business. (B) Home builders are not evenly distributed across the country. (C) The number of people who want second homes has been increasing each year for the past ten years. (D) Readers of House magazine are more likely than most people to want second homes. (E) House magazine includes articles about owning a second home as well as articles about building a second home. ii) Which of the following, if true, would undermine the validity of the investment advice in the paragraph above? (A) Some home owners are satisfied with only one home. (B) About half of the people who buy homes are investing in their first home. (C) About half of the people who buy homes have to take out a mortgage to do so. (D) Only a quarter of the homes that are built are sold within the first two weeks.

(E) Only a quarter of those who claim that they want a second home actually end up purchasing one. 30. Walter: A copy of an artwork should be worth exactly what the original is worth if the two works are visually indistinguishable. After all, if the two works are visually indistinguishable, they have all the same qualities, and if they have all the same qualities, their prices should be equal. Marissa: How little you understand art! Even if someone could make a perfect copy that is visually indistinguishable from the original, the copy would have a different history and hence not have all the same qualities as the original. i) Which of the following is a point at issue between Walter and Marissa? (A) Whether a copy of an artwork could ever be visually indistinguishable from the original (B) Whether the reproduction of a work of art is ever worth more than the original is worth (C) Whether a copy of a work of art is ever mistaken for the original (D) Whether a copy of a work of art could have all the same qualities as the original (E) Whether originality is the only valuable attribute that a work of art can possess ii) Marissa uses which of the following techniques in attempting to refute Walters argument? (A) Attacking his assumption that the price of an artwork indicates its worth (B) Raising a point that would undermine one of the claims on which his conclusion is based (C) Questioning his claim that a perfect copy of a work of art would be visually indistinguishable from the original (D) Giving reason to believe that Walter is unable to judge the quality of a work of art because of his inadequate understanding of the history of art (E) Proposing alternative criteria for determining whether two works of art are visually indistinguishable 31. The dean of computing must be respected by the academic staff and be competent to oversee the use of computers on campus. The only deans whom academics respect are those who hold doctoral degrees, and only someone who really knows about computers can competently oversee the use of computers on campus. Furthermore, the board of trustees has decided that the dean of computing must be selected from among this universitys staff. Therefore, the dean of computing must be a professor from this universitys computer science department. i) Which one of the following is an assumption on which the argument depends? (A) Academics respect only people who hold doctoral degrees. (B) All of these universitys professors have obtained doctoral degrees. (C) At this university, every professor who holds a doctoral degree in computer science really knows about computers. (D) All academics who hold doctoral degrees are respected by their academic colleagues. (E) Among this universitys staff members with doctoral degrees, only those in the computer science department really know about computers.

ii) Which one of the following statements, if true, would weaken the argument? (A) There are members of this universitys staff who hold doctoral degrees and who are not professors but who really know about computers. (B) There are members of this universitys philosophy department who do not hold doctoral degrees but who really know about computers. (C) Computer science professors who hold doctoral degrees but who are not members of this universitys staff have applied for the position of dean of computing. (D) Several members of the board of trustees of this university do not hold doctoral degrees. (E) Some members of the computer science department at this university are not respected by academics in other departments. 32. The company's coffee crop for 1998-99 totalled 8079 tonnes, an all time record. The increase over the previous year's production of 5830 tonnes was 38.58%. The previous highest crop was 6089 tonnes in 1970-71. The company had fixed a target of 8000 tonnes to be realised by the year 2000-01, and this has been achieved two years earlier, thanks to the emphasis laid on the key areas of irrigation, replacement of unproductive coffee bushes, intensive refilling and improved agricultural practices. It is now our endeavour to reach the target of 10000 tonnes in the year 2001-02. Which one of the following would contribute most to making the target of 10000 tonnes in 2001-02 unrealistic? The potential of the productivity enhancing measures implemented up to now has been exhausted. The total company land under coffee has remained constant since 1969 when an estate in the Nilgiri Hills was acquired. The sensitivity of the crop to climatic factors makes predictions about production uncertain. The target-setting procedures in the company have been proved to the sound by the achievement of the 8000 tonne target. 33. Animals in general are shrewd in proportion as they cultivate society. Elephants and beavers show the greatest signs of this sagacity when they are together in large numbers, but when man invades their communities they lose all their spirit of industry. Among insects, the labours of the bee and the ant have attracted the attention and admiration of naturalists, but all their sagacity seems to be lost upon separation, and a single bee or ant seems destitute of every degree of industry. It becomes the most stupid insect imaginable, and it languishes and soon dies. Which of the following can be inferred from the above passage? Humankind is responsible for the destruction of the natural habitat of animals and insects. Animals, in general, are unable to function effectively outside their normal social environment. Naturalists have great admiration for bees and ants, despite their lack of industry upon separation. Elephants and beavers are smarter than bees and ants in the presence of human beings. 34. In a recent report, the gross enrolment ratios at the primary level, that is, the number of children enrolled in classes one to five as a proportion of all children aged 6 to

10, were shown to be very high for most states; in many cases they were way above 100 percent! These figures are not worth anything, since they are based on the official enrolment data compiled from school records. They might as well stand for gross exaggeration ratios. Which of the following options best supports the claim that the ratios are exaggerated? The definition of gross enrolment ratio does not exclude, in its numerator, children below 6 years or above 10 years enrolled in classes one to five. A school attendance study found that many children enrolled in the school records were not meeting a minimum attendance requirement of 80 percent. A study estimated that close to 22 percent of children enrolled in the class one records were below 6 years of age and still to start going to school. Demographic surveys show shifts in the population profile which indicate that the number of children in the age group 6 to 10 years is declining 35. Automobile Dealer's Advertisement: The Highway Traffic Safety Institute reports that the PZ 1000 has the fewest injuries per accident of any car in its class. This shows that the PZ 1000 is one of the safest cars available today. Which of the following, if true, most seriously weakens the argument in the advertisement? (A) The Highway Traffic Safety Institute report listed many cars in other classes that had more injuries per accident than did the PZ 1000. (B) In recently years many more PZ 1000's have been sold than have any other kind of car in its class. (C) Cars in the class to which the PZ 1000 belongs are more likely to be involved in accidents than are other types of cars. (D) The difference between the number of injuries per accident for the PZ 1000 and that for other cars in its class is quite pronounced. (E) The Highway Traffic Safety Institute issues reports only once a year. 36. Animals in general are shrewd in proportion as they cultivate society. Elephants and beavers show the greatest signs of this sagacity when they are together in large numbers, but when man invades their communities they lose all their spirit of industry. Among insects, the labours of the bee and the ant have attracted the attention and admiration of naturalists, but all their sagacity seems to be lost upon separation, and a single bee or ant seems destitute of every degree of industry. It becomes the most stupid insect imaginable, and it languishes and soon dies. Which of the following can be inferred from the above passage? Humankind is responsible for the destruction of the natural habitat of animals and insects. Animals, in general, are unable to function effectively outside their normal social environment. Naturalists have great admiration for bees and ants, despite their lack of industry upon separation. Elephants and beavers are smarter than bees and ants in the presence of human beings.

37.A company that produces mens cologne had been advertising the product in generalcirculation magazines for several years. Then one year the company decided to advertise its cologne exclusively in those sports magazines with a predominantly male readership. That year the company sold fewer bottles of cologne than it had in any of the three immediately preceding years. Which one of the following, if true, best helps to explain why the sale of the companys cologne dropped that year? (A) Television advertising reaches more people than does magazine advertising, but the company never advertised its cologne on television because of the high cost. (B) The general-circulation magazines in which the company had placed its advertisements experienced a large rise in circulation recently. (C) Most men do not wear cologne on a regular basis. (D) Women often buy cologne as gifts for male friends or relatives. (E) Successful advertisements for mens cologne often feature well-known athletes. 38. Kim: The rapidly growing world population is increasing demands on food producers in ways that threaten our natural resources. With more land needed for both food production and urban areas, less land will be available for forests and wildlife habitats. Hampton: You are overlooking the promises of technology. I am confident that improvements in agriculture will allow us to feed the world population of ten billion predicted for 2050 without significantly increasing the percentage of the worlds land now devoted to agriculture. Kims and Hamptons statements most strongly support the claim that both of them would agree with which one of the following? (A) Efforts should be taken to slow the rate of human population growth and to increase the amount of land committed to agriculture. (B) Continued research into more-efficient agricultural practices and innovative biotechnology aimed at producing more food on less land would be beneficial. (C) Agricultural and wilderness areas need to be protected from urban encroachment by preparing urban areas for greater population density. (D) In the next half century, human population growth will continue to erode wildlife habitats and diminish forests. (E) The human diet needs to be modified in the next half century because of the depletion of our natural resources due to overpopulation. 39. Following his 1826 expedition in Oceania, French navigator Jules Dumont d'Urville invented the terms Malaysia, Micronesia and Melanesia, distinguishing these Pacific cultures and island groups from the already existing term Polynesia. In 1831, he proposed these terms to The Socit de Gographie. Dumont d'Urville described Malaysia as "an area commonly known as the East Indies". In 1850, the English ethnologist George Samuel Windsor Earl, writing in the Journal of the Indian Archipelago and Eastern Asia, proposed naming the islands of Southeast Asia as Melayunesia or Indunesia, favouring the former. Which of the following is an appropriate title for the passage? 1)The Adventures of Jules Dumont d'Urville, French Navigator 2)The culmination of the The Socit de Gographie into a popular governing body

3)The etymology of Malaysia and other surrounding nations 4)How the East Indies became Malaysia 5)The Islands of South East Asia 40.The importance of the ozone layer to terrestrial animals is that it entirely filters out some wavelengths of light but lets others through. Holes in the ozone layer and the dangers associated with these holes are well documented. However, one danger that has not been given sufficient attention is that these holes could lead to severe eye damage for animals of many species. Which one of the following is most strongly supported by the statements above, if they are true? (A) All wavelengths of sunlight that can cause eye damage are filtered out by the ozone layer where it is intact. (B) Few species of animals live on a part of the earth's surface that is not threatened by holes in the ozone layer. (C) Some species of animals have eyes that will not suffer any damage when exposed to unfiltered sunlight. (D) A single wavelength of sunlight can cause severe damage to the eyes of most species of animals. (E) Some wavelengths of sunlight that cause eye damage are more likely to reach the earth's surface where there are holes in the ozone layer than where there are not. 41.In an experiment, two-year-old boys and their fathers made pie dough together using rolling pins and other utensils. Each father-son pair used a rolling pin that was distinctively different from those used by the other father-son pairs, and each father repeated the phrase "rolling pin" each time his son used it. But when the children were asked to identify all of the rolling pins among a group of kitchen utensils that included several rolling pins, each child picked only the one that he had used. Which one of the following inferences is most supported by the information above? (A) The children did not grasp the function of a rolling pin. (B) No two children understood the name "rolling pin" to apply to the same object. (C) The children understood that all rolling pins have the same general shape. (D) Each child was able to identify correctly only the utensils that he had used. (E) The children were not able to distinguish the rolling pins they used from other rolling pins. 42. Some environmentalists question the prudence of exploiting features of the environment, arguing that there are no economic benefits to be gained from forests, mountains, or wetlands that no longer exist. Many environmentalists claim that because nature has intrinsic value it would be wrong to destroy such features of the environment, even if the economic costs of doing so were outweighed by the economic costs of not doing so. Which one of the following can be logically inferred from the passage? (A) It is economically imprudent to exploit features of the environment. (B) Some environmentalists appeal to a noneconomic justification in questioning the defensibility of exploiting features of the environment. (C) Most environmentalists appeal to economic reasons in questioning the defensibility of exploiting features of the environment. (D) Many environmentalists provide only a noneconomic justification in questioning the defensibility of exploiting features of the environment.

(E) Even if there is no economic reason for protecting the environment, there is a sound noneconomic justification for doing so. 43. In 1990 all of the people who applied for a job at Evco also applied for a job at Radeco, and Evco and Radeco each offered jobs to half of these applicants. Therefore, every one of these applicants must have been offered a job in 1990. The argument above is based on which of the following assumptions about these job applicants? (A) All of the applicants were very well qualified for a job at either Evco or Radeco. (B) All of the applicants accepted a job at either Evco or Radeco. (C) None of the applicants was offered a job by both Evco and Radeco. (D) None of the applicants had applied for jobs at places other than Evco and Radeco. (E) None of the applicants had previously worked for either Evco or Radeco. 44. The geese that gather at the pond of a large corporation create a hazard for executives who use the corporate helicopter, whose landing site is 40 feet away from the pond. To solve the problem, the corporation plans to import a large number of herding dogs to keep the geese away from the helicopter. Which of the following, if a realistic possibility, would cast the most serious doubt on the prospects for success of the corporations plan? (A) The dogs will form an uncontrollable pack. (B) The dogs will require training to learn to herd the geese. (C) The dogs will frighten away foxes that prey on old and sick geese. (D) It will be necessary to keep the dogs in quarantine for 30 days after importing them. (E) Some of the geese will move to the pond of another corporation in order to avoid being herded by the dogs. 45. Whenever a major airplane accident occurs, there is a dramatic increase in the number of airplane mishaps reported, a phenomenon that may last for as long as a few months after the accident. Airline officials assert that the publicity given the gruesomeness of major airplane accidents focuses media attention on the airline industry and the increase in the number of reported accidents is caused by an increase in the number of news sources covering airline accident, not by an increase in the number of accidents. Which of the following, if true, would seriously weaken the assertions of the airline officials? (A) The publicity surrounding airline accidents is largely limited to the country in which the crash occurred. (B) Airline accidents tend to occur far more often during certain peak travel months. (C) News organizations do not have any guidelines to help them decide how severe or how close an accident must be for it to receive coverage. (D) Airplane accidents receive coverage by news sources only when the news sources find it advantageous to do so. (E) Studies by government regulations show that the number of airplane flight miles remains relatively constant from month to month. 46. Critics of sales seminars run by outside consultants point out that since 1987, revenues of vacuum cleaner companies whose employees attended consultant-led seminars were lower than revenues of vacuum cleaner companies whose employees did not attend such seminars. The critics charge that for vacuum cleaner companies, the

sales seminars are ill conceived and a waste of money. Which of the following, if true, is the most effective challenge to the critics of sales seminars? (A) Those vacuum cleaner companies whose sales were highest prior to 1987 are the only companies that did not send employees to the seminars. (B) Vacuum cleaner companies that have sent employees to sales seminars since 1987 experienced a greater drop in sales than they had prior to 1987. (C) The cost of vacuum cleaner sales seminars run by outside consultants has risen dramatically since 1987. (D) The poor design of vacuum cleaner sales seminars is not the only reason for their ineffectiveness. (E) Since 1987, sales of vacuum cleaners have risen twenty percent. 47. Although statistics and definitions are inexact, educated guesses put the number of refugees worldwide at well over 10 million. The overwhelming majority prefer to return to their native land than to emigrate to a foreign one. The millions of refugees from Afghanistan are sufficient proof: Despite the toll the war and subsequent fighting have taken on their country, very few have applied for permission to emigrate. Which of the following, if true, would most strengthen the argument above? (A) Most refugees are as reluctant to emigrate as are the refugees from Afghanistan. (B) The refugees from Afghanistan fled what they considered political oppression rather than economic disaster. (C) Most of the children born to refugees prefer to remain in their adoptive country rather than return to the land their parents left. (D) Although refugees flee their homes for a variety of different reasons, the overwhelming majority are looking for improved living conditions. (E) The number of refugees worldwide has risen dramatically over the last ten years. 48. Recent statistics make it seem unlikely that the total consumption of electricity determines its cost to individual consumers. Recent increases in total consumption, especially during the hot summer months, have sometimes been accompanied by a decrease in the cost per unit and at other times by an increase. Which of the following positions is best supported by the information presented above? (A) It must be the case that the cost of electricity to consumers is what determines the total consumption. (B) Even though a correlation exists between the total consumption of electricity and its cost to consumers, no causal relation exists. (C) Further investigation into the way these statistics were gathered is certainly called for. (D) The cost of electricity depends upon something other than the total consumption of electricity. (E) The cost-per-unit of electricity to consumers is dependent on the total electricity consumption. 49. Mr. Janeck: I dont believe Stevenson will win the election for governor. Few voters are willing to elect a businessman with no political experience to such a responsible

public office. Ms. Siuzdak: Youre wrong. The experience of running a major corporation is a valuable preparation for the task of running a state government. M. Siuzdaks response shows that she has interpreted Mr. Janecks remark to imply which of the following? (A) Mr. Janeck considers Stevenson unqualified for the office of governor. (B) No candidate without political experience has ever been elected governor of a state. (C) Mr. Janeck believes that political leadership and business leadership are closely analogous. (D) A career spent in the pursuit of profit can be an impediment to ones ability to run a state government fairly. (E) Voters generally overestimate the value of political experience when selecting a candidate. 50. Some educators claim that it is best that school courses cover only basic subject matter, but cover it in depth. These educators argue that if student achieve a solid grasp of the basic concepts and investigatory techniques in a subject, they will be able to explore the breadth of that subject on their own after the course is over. But if they simply learn a lot of factual information, without truly understanding its significance, they will not be well equipped for further study on their own. The educators reasoning provide grounds for accepting which one of the following statements? (A) It is easier to understand how plants and animals are classified after learning how plants and animals can be useful . (B) It is more difficult to recall the details of a dull and complicated lecture than of a lively and interesting one. (C) It is easier to remember new ideas explained personally by a teacher than ideas that one explores independently. (D) It is easier to understand any Greek tragedy after one has analyzed a few of them in detail. (E) It is easier to learn many simple ideas well than to learn a few complicated ideas well. 51. Damming the Merv River would provide irrigation for the dry land in its upstream areas; unfortunately, a dam would reduce agricultural productivity in the fertile land downstream by reducing the availability and quality of the water there. The productivity loss in the downstream area would be greater than the productivity gain upstream , so building a dam would yield no overall gain in agricultural productivity in the region as a whole. The reasoning in the argument above most closely parallels that in which one of the following? (A) disease-causing bacteria in eggs can be destroyed by overcooking the eggs, but the eggs then become much less appetizing; health is more important than taste, however, so it is better to overcook eggs than not to do so. (B) Increasing the price of transatlantic telephone calls will discourage many private individuals from making them. But since most transatlantic telephone calls are made by businesses, not by private individuals, a rate increase will not reduce telephone company profits.

(C) A new highway will allow suburban commuters to reach the city more quickly, but not without causing increased delays within the city that will more than offset any time saved on the highway. Therefore, the highway will not educe suburban commuters overall commuting time. (D) Doctors can prescribe antibiotics fro many minor illnesses, but antibiotics are expensive, and these illnesses can often be cured by rest alone. Therefore, it is better to rest at home than to see a doctor for these illnesses. (E) A certain chemical will kill garden pest that damage tomatoes, but that chemical will damage certain other plants more severely than the pests damage the tomatoes, so the only garden that will benefit from the use of the chemical are those in which only tomatoes are grown. 52. Some critics argue that an operas stage directions are never reflected in its music. Many comic scenes in Mozarts operas, however, open with violin phrases that sound like the squeaking of changing scenery. Clearly Mozart intended the music to echo the sounds occurring while stage directions are carried out. Hence, a change of scenerythe most basic and frequent stage directioncan be reflected in the music, which means that other operatic stage directions can be as well. In the argument, the statement that many comic scenes in Mozarts operas open with violin phrases that sound like the squeaking of changing scenery is offered in support of the claim that (A) a change of scenery is the stage direction most frequently reflected in an operas music (B) an operas stage directions are never reflected in its music (C) an operas music can have an effect on the operas stage directions (D) a variety of stage directions can be reflected in an operas music (E) the most frequent relation between an operas music and its stage directions is one of musical imitation of the sounds that occur when a direction is carried out 53. Lecture: Given our current state of knowledge and technology, we can say that the generalization that the entropy of a closed system cannot decrease for any spontaneous process has not been falsified by any of our tests of that generalization. So we conclude it to be true universally. Yet, it must be admitted that this generalization has not been conclusively verified, in the sense that it has not been tested in every corner of the universe, under every feasible condition. Nevertheless, this generalization is correctly regarded as a scientific law; indeed, it is referred to as the Second Law of Thermodynamics. Which one of the following principles, if valid, most justifies the lecturers classification of the generalization described above? (A) Whatever is a scientific law has not been falsified. (B) If a generalization is confirmed only under a few circumstances, it should not be considered a scientific law. (C) Whatever is true universally will eventually be confirmed to the extent current science allows. (D) If a generalization is confirmed to the extent current science allows, then it is considered a scientific law. (E) Whatever is regarded as a scientific law will eventually be conclusively verified.

54. Bureaucratic mechanisms are engineered to resist change. Thus, despite growing dissatisfaction with complex bureaucratic systems, it is unlikely that bureaucracies will be simplified. The claim that bureaucratic mechanisms are engineered to resist change plays which one of the following roles in the argument? (A) It is a premise offered in support of the claim that it is unlikely that bureaucracies will be simplified. (B) It is a conclusion for which the only support offered is the claim that dissatisfaction with complex bureaucratic systems is growing. (C) It is cited as evidence that bureaucratic systems are becoming more and more complex. (D) It is used to weaken the claim that bureaucracies should be simplified. (E) It is a conclusion for which the claim that bureaucracies are unlikely to be simplified is offered as support. 55. In speech, when words or sentences are ambiguous, gesture and tone of voice are used to indicate the intended meaning. Writers, of course, cannot use gesture or tone of voice and must rely instead on style; the reader detects the writers intention from the arrangement of words and sentences. Which one of the following statements is most strongly supported by the information above? (A) The primary function of style in writing is to augment the literal meanings of the words and sentences used. (B) The intended meaning of a piece of writing is indicated in part by the writers arrangement of words and sentences. (C) It is easier for a listener to detect the tone of a speaker than for a reader to detect the style of the writer. (D) A writers intention will always be interpreted differently by different readers. (E) The writers arrangement of words and sentences completely determines the aesthetic value of his or her writing. 56. Last year a large firm set a goal of decreasing its workforce by 25 percent. Three divisions, totaling 25 percent of its workforce at that time, were to be eliminated and no new people hired. These divisions have since been eliminated and no new people have joined the firm, but its workforce has decreased by only 15 percent. Which one of the following, if true, contributes most to an explanation of the difference in the planned versus the actual reduction in the workforce? (A) The three divisions that were eliminated were well run and had the potential to earn profits. (B) Normal attrition in the retained divisions continued to reduce staff because no new people were added to the firm. (C) Some of the employees in the eliminated divisions were eligible for early retirement and chose that option. (D) As the divisions were being eliminated some of their employees were assigned to other divisions. (E) Employees in the retained divisions were forced to work faster to offset the loss of the eliminated divisions.

57.4 One of the advantages of Bacillus thuringiensls (B.t.) toxins over chemical insecticides results from their specificity for pest insects. The toxins have no known detrimental effects on mammals or birds. In addition, the limited range of activity of the toxins toward insects means that often a particular toxin will kill pest species but not affect insets that prey upon the species. This advantage makes B.t. toxins preferable to chemical insecticides for use as components of insect pest management programs. Which one of the following statements, if true, most weakens the argument? (A) Chemical insecticides cause harm to a greater number of insect species than do B.t. toxins. (B) No particular B.t. toxin is effective against all insects. (C) B.t. toxins do not harm weeds that do damage to farm crops. (D) Insects build up resistance more readily to B.t. toxins than to chemical insecticides. (E) Birds and rodents often do greater damage to farm crops than do insects. 58. Flavonoids are a common component of almost all plants, but a specific variety of flavonoid in apples has been found to be an antioxidant. Antioxidants are known to be a factor in the prevention of heart disease. Which one of the following can be properly inferred from the passage? (A) A diet composed largely of fruits and vegetables will help to prevent heart disease. (B) Flavonoids are essential to preventing heart disease. (C) Eating at least one apple each day will prevent heart disease. (D) At least one type of flavonoid helps to prevent heart disease 59. An experimental microwave clothes dryer heats neither air nor cloth. Rather, it heats water on clothes, thereby saving electricity and protecting delicate fibers by operating at a lower temperature. Microwaves are waves that usually heat metal objects, but developers of a microwave dryer are perfecting a process that will prevent thin metal objects such as hairpins from heating up and burning clothes. Which of the following, if true, most strongly indicates that the process, when perfected, will be insufficient to make the dryer readily marketable? (A) Metal snap fasteners on clothes that are commonly put into drying machines are about the same thickness as most hairpins. (B) Many clothes that are currently placed into mechanical dryers are not placed there along with hairpins or other thin metal objects. (C) The experimental microwave dryer uses more electricity than future, improved models would be expected to use. (D) Drying clothes with the process would not cause more shrinkage than the currently used mechanical drying process causes. (E) Many clothes that are frequently machine-dried by prospective customers incorporate thick metal parts such as decorative brass studs or buttons. 60. We doubt that the latest government report will scare Americans away from ham, bacon, sausages, hot dogs, bologna, and salami or that it will empty out the bars or cause a run on natural food supplies. If a diet were to be mandated from Washington,

Americans probably would order the exact opposite course. Therefore, the diet that does make sense is to eat a balanced and varied diet composed of foods from all food groups and containing a reasonable caloric intake. Which of the following is (are) specifically implied by the passage? I. Vitamins are necessary to combat disease. II. A recent report warned of the risks of meat and alcoholic beverages. III. Unorthodox suggestions for a more nutritional diet were recently made by the government. A. I only B. II only C. III only D. I and II only E. II and III only 61.The new vehicle inspection program is needed to protect the quality of the state's air, for us and for our children. Auto exhausts are a leading contributor to coughing, wheezing, choking, and pollution. The state's long-term interests in the health of its citizens and in this area as a place to live, work, and conduct business depend on clean air. Which of the following is an unstated assumption made by the author? A. Working and conducting business may be different activities. B. The Minister believes that Indian companies are looking for business expansion.The state has been interested in the health of its citizens even before this inspection program was proposed. C. Exhaust emissions contribute to pollution. D. The new inspection program will be effective. E. Our ancestors did not suffer from air pollution 62.Which of the following is the most logical completion of the passage below? In the 1940s, the introduction of the 33 rpm long-playing vinyl record completely changed the way we listen to music. The breakable and three minute 78 rpm record soon disappeared from the marketplace. In our day, the compact disk, superior in quality and convenience, has replaced the vinyl long-playing record and will . . . A. increase the size of the record-buying public. B. increases the profits of the record industry. C. drives the 78 rpm record from the second-hand market. D. makes the manufacture of phonographs that play 33 rpm records unnecessary. E. encourage the growth of computer-generated music 63.In most economies, the government plays a role in the market system. Governments enforce the "rules of the game," impose taxes, and may control prices through price ceilings or price supports. These actions necessarily may create shortages or surpluses. In most developed and interdependent economies, the necessity of the government's

playing some role in the economy is disputed. The author of the passage would probably agree that A. economic surpluses are always good. B. market shortages are a necessary evil. C. higher prices strengthen the economy. D. price ceilings add to the shortages. E. surpluses are not usually created intentionally 64.That the policy of nuclear deterrence has worked thus far is unquestionable. Since the end of the Second World War, the very fact that there were nuclear armaments in existence has kept major powers from using nuclear weapons, for fear of starting a worldwide nuclear exchange that would make the land of the power initiating it uninhabitable. The proof is that a third world war between superpowers has not happened. Which one of the following, if true, indicates a flaw in the argument? A. Maintaining a high level of nuclear armaments represents a significant drain on a country's economy. B. From what has happened in the past, it is impossible to infer with certainty what will happen in the future, so an accident could still trigger a third world war between superpowers. C. Continuing to produce nuclear weapons beyond the minimum needed for deterrence increases the likelihood of a nuclear accident. D. The major powers have engaged in many smaller-scale military operations since the end of the Second World War, while refraining from a nuclear confrontation. E. It cannot be known whether it was nuclear deterrence that worked, or some other factor, such as recognition of the economic value of remaining at peace 65. With Proposition 13, if you bought your house 11 years ago for $75,000, your property tax would be approximately $914 a year (1 percent of $75,000 increased by 2 percent each year for 11 years); and if your neighbour bought an identical house next door to you for $200,000 this year, his tax would be $2,000 (1 percent of $200,000). Without Proposition 13, both you and your neighbour would pay $6,000 a year in property taxes (3 percent of $200,000). Which of the following is the conclusion for which the author most likely is arguing in the passage above? A. Proposition 13 is unconstitutional because it imposes an unequal tax on properties of equal value. B. If Proposition 13 is repealed; every homeowner is likely to experience a substantial increase in property taxes. C. By preventing inflation from driving up property values, Proposition 13 has saved homeowners thousands of dollars in property taxes. D. If Proposition 13 is not repealed; identical properties will continue to be taxed at different rates. E. Proposition 13 has benefited some homeowners more than others.

66. The problem with arms reduction is that it is an illusory concept benefitting none. Even though designated stockpiles are being reduced, the weapons race continues, as the destructive power of new technologies and remaining arsenals is enhanced in order to maintain the pre-existing firepower. Thus, although it fosters an illusion of progress, arms reduction does nothing to curtail the proliferation of weaponry, and all must continue to live under the constant threat of annihilation. Which one of the following, if true, would most strengthen the author's argument? A) Arms reduction allows steady maintenance of the existing balance of power. B) No arms limitation proposals have aimed at completely eliminating a nation's armament stockpile. C) The five largest military powers have increased funding for new weapons in each of the last ten years. D) The distinction between offensive and defensive weapon systems is often merely a matter of interpretation. E) Arms limitation treaties have only accounted for the elimination of fifteen percent of the total firepower possessed by the five largest military powers. 67. With the proliferation of private companies that specialize in genetic research, it was feared that they would impose silence on the results of in-house research. This constraint, in turn, would slow the progress of genetic engineering to alleviate patient suffering. Which of the following, if true, most seriously weakens the prediction of scientific secrecy above? A. Genetic research funded by industry has reached some conclusions that are of major scientific importance. B. When the results of scientific research are kept secret, independent researcher are unable to build on those results. C. To enhance the companies standing in the community, the genetic companies encourage employees to publish their results, especially results that are important. D. Since the research priorities of companies that specialize in genetic research are not the same as those of academic institutions, the financial support of research by such companies distorts the research agenda E. Companies that specialize in genetic research devote some of their research resources to problems that are of fundamental scientific importance and that are not expected to produce immediate practical applications. 68. But because the idea of private property has been permitted to override with its selfishness, the common good of humanity it does not follow that there are not limits within which that idea can function for the general convenience and advantage. Which of the following is most likely to weaken the argument? (1) All the people of the society should progress at an equitable rate and there should be no disparities and private property does bring about a tremendous disparity. (2) One should not strive for the common good of humanity at all, instead one should be concerned with maximising one's own wealth.

(3) One should learn from the experiences of former communist nations and should not repeat his mistakes at all. (4) Even prosperous capitalist countries like the USA have their share of social problems. 69. Anyone who supports the Telangana state has no chance of securing a majority in the Andhra Pradesh state legislature. Also, anyone who understands the nitty-gritty of regional development would not support carving out the state of Telangana. So, only someone who truly understands regional development would have any chance of securing a majority in the house. The reasoning in the above argument is flawed because: a)A person who understands the nitty-gritty of regional development could also have a chance of securing a majority in the state legislature. b) A person who does not understand the nitty-gritty of regional development could also support the creation of the Telangana state. c) A person who does not understand the nitty-gritty of regional development may also not support the creation of the Telangana state. d)A person who does not understand the nitty-gritty of regional development can have no chance of securing a majority in the state legislature. 70. The spat between Reliance Communications and Reliance Petrochemical has taken a new turn. Mukesh Ambani, owner of Reliance Petrochemical, has stated that he will supply natural gas to Anil Ambani owned gas power projects only when Anil Ambani owned Reliance Communications shelves its plan to sell the shares of Reliance Communications. So, the shares of Reliance Communications will not be sold. Which of the following is the underlying assumption in the above argument? a) Anil Ambani will not explore any other options available to him. b) There are no other natural gas providers for the power projects owned by Anil Ambani. c) Anil Ambani will accept the proposal offered by Mukesh Ambani. d) Mukesh Ambani will not review the conditions offered in his proposal. 71. A news channel reporter raised a relevant question when a famous Astrologer referred to instances when his prophecies came true. The reporter asked, "But what about those prophecies that turned out to be incorrect?" We have a tendency to see what we wish to see and neglect everything that goes contrary to what we wish to see. Which one of the following is the best example of the error of reasoning described by the author in the paragraph? a)December is the unluckiest month of a year. Just this past December, I not only lost

my job but also a dear friend of mine. b)I can no longer consider my lucky TV lucky. On the last ten occasions when I watched an India-Pakistan match on my TV, India lost the match. c)I have appeared for the job interview and cleared the same, but I am not sure what is my lucky charm. d)Every year around 10% students of IIM Ahmedabad opt out of placements to start their own ventures. So, we can assume that this year IIM Ahmedabad will place around 90% of its students. 72. Although fullerenes - spherical molecules made entirely of carbon - were first found in the laboratory, they have since been found in nature, formed in fissures of the rare mineral shungite. Since laboratory synthesis of fullerenes requires distinctive conditions of temperature and pressure, this discovery should give geologists a test case for evaluating hypothesis about the state of the Earth's crust at the time these naturally occurring fullerenes were formed. Which of the following, if true, most seriously undermines the argument? A. Confirming that the shungite genuinely contained fullerenes took careful experimentation B. Some fullerenes have also been found on the remains of a small meteorite that collided with a spacecraft. C. The mineral shungite itself contains large amounts of carbon, from which the fullerenes apparently formed. D. The naturally occurring fullerenes are arranged in a previously unknown crystalline structure E. Shungite itself is formed only under distinctive conditions. 73. Insect Infestations in certain cotton growing regions of the world have caused dramatic increases of cotton on the world market. Knowing that cotton plants mature quickly, many soybean growers in Ortovia plan to cease growing soybeans, the price of which has long been stable and to begin raising cotton instead, thereby taking advantage of the high price of cotton to increase their income significantly over the next several years. Which of the following, if true, most calls into question the reasoning on which the plan is based? A. The cost of raising soybeans has increased significantly over the past several years and is expected to continue to climb. B. Tests of a newly developed, inexpensive pesticide have shown it to be both environmentally safe and effective against the insects that have infected the cotton crops. C. In the past several years, there has been no sharp increase in the demand for cotton, and for goods made out of cotton. D. Many consumers consider cotton cloth a necessity rather than a luxury and would be willing to pay significantly higher prices for cotton goods than they are currently paying E. The species of insect that has infested the cotton plants has never been known to infest soybean plants

74. At an enormous research cost, a leading chemical company has developed a manufacturing process for converting wood fibers into a plastic. According to the company, this new plastic can be used for, among other things, the hulls of small sailboats. But what does the company think sailboat hulls used to be made of? Surely the mania for high technology can scarcely go further than this. i)The author's opinion of the manufacturing process described in the passage is based primarily on the fact that (A) plastic is unlikely to be durable enough for high-quality sailboat hulls (B) the research costs of developing the process outweigh any savings possible from the use of the plastic (C) a small sailboat is not normally regarded as a high-tech product (D) hulls for small sailboats can be made from wood without converting it into plastic (E) many other spheres of human activity are in far greater need of technological research ii)Which of the following, if true, would most seriously weaken the author's conclusion? (A) The plastic produced by the process is considerably lighter, stronger, and more watertight than wood. (B) The wood used in producing the plastic is itself in increasingly short supply. (C) The cost of the manufacturing process of the plastic increases the cost of producing a sailboat hull by 10 to 15 percent. (D) Much of the cost of the research that developed the new process will be written off for tax purposes by the chemical company. (E) The development of the new plastic is expected to help make the chemical company an important supplier of boat-building materials. 75. A young man eager to become a master swordsman journeyed to the home of the greatest teacher of swordsmanship in the kingdom. He asked the teacher, "How quickly can you teach me to be a master swordsman?" The old teacher replied, "It will take ten years." Unsatisfied, the young man asked, "What if I am willing to work night and day, every day of the year?" the teacher replied, "In that case, it will take twenty years." The teacher's main point is that an important quality of a master swordsman is (A) humility (B) willingness to work hard (C) respect for one's elders (D) patience (E) determination 76. In an experiment, two different types of recorded music were played for neonates in adjacent nurseries in a hospital. In nursery A, classical music was played; in nursery B, rock music was played. After two weeks, it was found that the babies in nursery A cried less, suffered fewer minor ailments, and gained more weight than did the babies in nursery B. i) In evaluating the validity of the conclusion suggested by the experiment above, it would be most important to know which of the following? (A) The musical preferences of the parents of the two groups of newborns (B) Whether the newborns in both nurseries were equally healthy and happy at the start of the experiment (C) Whether loud rock music can damage the hearing of newborns

(D) What the average weight of the neonates was before and after the experiment (E) Whether the music was played in the nurseries at all times or only at certain times ii) Which of the following additional experimental data would support the hypothesis that classical music is beneficial to the development of newborn? (A) The neonates in a nursery where no music was played fared better than those in nursery B. (B) Nursery A contained 15 percent more premature babies than nursery B. (C) The newborns in nursery A cried less, suffered fewer minor ailments, and gained more weight than did newborns in a nursery with no music. (D) The music played in nursery A was louder than that played in nursery B. (E) The ratio of nurses to newborns in nursery B was 1 to 4; in nursery A, it was 1 to 6. 77. The ancient city of Cephesa was not buried by an eruption of Mt. Amnos in A.D. 310, as some believe. The eruption in the year 310 damaged the city, but it did not destroy it. Cephesa survived for another century before it finally met its destruction in another eruption around A.D. 415. Which of the following, if true, would most strengthen the author's claim that the city of Cephesa was not buried by the eruption of Mt. Amnos in A.D. 310? (A) The city of Cephesa is mentioned in a historical work known to have been written in A.D. 400. (B) Coins bearing the image of an emperor who lived around A.D. 410 have been discovered in the ruins of Cephesa, which were preserved by the cinders and ashes that buried the city. (C) Geological evidence shows that the eruption of Mt. Amnos in A.D. 415 deposited a 10-foot-thick layer of lava on the city of Cephesa. (D) Artworks from the city of Cephesa have been found in the ruins of another city known to have been destroyed in A.D. 420. (E) A historical work written in A.D. 430 refers to the eruption of Mt. Amnos in A.D. 415. 78. Opponents of laws that require automobile drivers and passengers to wear seat belts argue that in a free society people have the right to take risks as long as the people do not harm other as a result of taking the risks. As a result, they conclude that it should be each persons decision whether or not to wear a seat belt. Which of the following, if true, most seriously weakens the conclusion drawn above? A. Many new cars are built with seat belts that automatically fasten when someone sits in the front seat. B. Automobile insurance rates for all automobile owners are higher because of the need to pay for the increased injuries or deaths of people not wearing seat belts. C. Passengers in airplanes are required to wear seat belts during takeoffs and landings. D. The rate of automobile fatalities in states that do not have mandatory seat belt laws is greater than the rate of fatalities in states that do have such laws. E. In automobile accidents, a greater number of passengers who do not wear seat belts are injured than are passengers who do wear seat belts. 79.The cost of producing radios in Country Q is ten percent less than the cost of producing radios in Country Y. even after transportation fees and tariff charges are added, it is still cheaper for a company to import radios from Country Q to Country Y

than to produce radios in Country Y. The statements above, if true, best support which of the following assertions? A. labor costs in Country Q are ten percent below those in Country Y. B. importing radios from Country Q to Country Y will eliminate ten percent of the manufacturing jobs in Country Y. C. the tariff on a radio imported from Country Q to Country Y is less than ten percent of the cost of manufacturing the radio in Country Y. D. the fee for transporting a radio from Country Q to Country Y is more than ten percent of the cost of manufacturing the radio in Country Q. E. it takes ten percent less time to manufacture a radios in Country Q than it does in Country Y. 80. During the Second World War, about 375,000 civilians died in the United States and about 408,000 members of the United States armed forces died overseas. On the basis the those figures, it can be concluded that it was not much more dangerous to be overseas in the armed forces during the Second World War than it was to stay at home as a civilian. Which of the following would reveal most clearly the absurdity of the conclusion drawn above? A. Counting deaths among members of the armed forces who served in the United State in addition to deaths among members of the armed forces serving overseas B. Expressing the difference between the numbers of deaths among civilians and members of the armed forces as a percentage of the total number of deaths C. Separating deaths caused by accidents during service in the armed forces from deaths caused by combat injuries D. Comparing death rates per thousand members of each group rather than comparing total numbers of deaths E. Comparing deaths caused by accidents in the United States to deaths caused by combat in the armed forces 81. Toughened hiring standards have not been the primary cause of the present staffing shortage in public schools. The shortage of teachers is primarily caused by the fact that in recent years teachers have not experienced any improvements in working conditions and their salaries have not kept pace with salaries in other professions. Which of the following, if true, would most support the claims above? A. Many teachers already in the profession would not have been hired under the new hiring standards. B. Today more teachers are entering the profession with a higher educational level than in the past. C. Some teachers have cited higher standards for hiring as a reason for the current staffing shortage. D. Many teachers have cited low pay and lack of professional freedom as reasons for their leaving the profession. E. Many prospective teachers have cited the new hiring standards as a reason for not entering the profession.

82. A proposed ordinance requires the installation in new homes of sprinklers automatically triggered by the presence of a fire. However, a home builder argued that because more than ninety percent of residential fires are extinguished by a household member, residential sprinklers would only marginally decrease property damage caused by residential fires. Which of the following, if true, would most seriously weaken the home builder's argument? A. most individuals have no formal training in how to extinguish fires. B. Since new homes are only a tiny percentage of available housing in the city, the new ordinance would be extremely narrow in scope. C. The installation of smoke detectors in new residences costs significantly less than the installation of sprinklers. D. In the city where the ordinance was proposed, the average time required by the fire department to respond to a fire was less than the national average. E. The largest proportion of property damage that results from residential fires is caused by fires that start when no household member is present. 83. Even though most universities retain the royalties from faculty members' inventions, the faculty members retain the royalties from books and articles they write. Therefore, faculty members should retain the royalties from the educational computer software they develop. The conclusion above would be more reasonably drawn if which of the following were inserted into the argument as an additional premise? A. Royalties from inventions are higher than royalties from educational software programs. B. Faculty members are more likely to produce educational software programs than inventions. C. Inventions bring more prestige to universities that do books and articles. D. In the experience of most universities, educational software programs are more marketable that are books and articles. E. In terms of the criteria used to award royalties, educational software programs are more nearly comparable to books and articles than to inventions. 84. Increase in the level of high-density lipoprotein (HDL) in the human bloodstream lower bloodstream-cholesterol levels by increasing the body's capacity to rid itself of excess cholesterol. Levels of HDL in the bloodstream of some individuals are significantly increased by a program of regular exercise and weight reduction. Which of the following can be correctly inferred from the statements above? A. Individuals who are underweight do not run any risk of developing high levels of cholesterol in the bloodstream. B. Individuals who do not exercise regularly have a high risk of developing high levels of cholesterol in the bloodstream late in life. C. Exercise and weight reduction are the most effective methods of lowering bloodstream cholesterol levels in humans. D. A program of regular exercise and weight reduction lowers cholesterol levels in the

bloodstream of some individuals. E. Only regular exercise is necessary to decrease cholesterol levels in the bloodstream of individuals of average weight. 85. When limitations were in effect on nuclear-arms testing, people tended to save more of their money, but when nuclear-arms testing increased, people tended to spend more of their money. The perceived threat of nuclear catastrophe, therefore, decreases the willingness of people to postpone consumption for the sake of saving money. The argument above assumes that A. the perceived threat of nuclear catastrophe has increased over the years. B. most people supported the development of nuclear arms C. people's perception of the threat of nuclear catastrophe depends on the amount of nuclear-arms testing being done D. the people who saved the most money when nuclear-arms testing was limited were the ones who supported such limitations E. there are more consumer goods available when nuclear-arms testing increases 86. In parts of South America, vitamin-A deficiency is a serious health problem, especially among children. In one region, agriculturists hope to improve nutrition by encouraging farmers to plant a new variety of sweet potato called SPK004 that is rich in betacarotene, which the body converts into vitamin A. The plan has good chances of success, since sweet potato is a staple of the region's diet and agriculture, and the varieties currently grown contain little beta-carotene. Which of the following, if true, most strongly supports the prediction that the plan will succeed? A. There are other vegetables currently grown in the region that contain more betacarotene than the currently cultivated varieties of sweet potato do. B. The flesh of SPK004 differs from that of the currently cultivated sweet potatoes in colors and textures, so traditional foods would look somewhat different when prepared from SPK004. C. For successful cultivation of SPK004, a soil significantly richer in nitrogen is needed than is needed for the varieties of sweet potato currently cultivated in the region. D. There are no other varieties of sweet potato that are significantly richer in betacarotene than SPK004 is. E. The currently cultivated varieties of sweet potato contain no important nutrients that SPK004 lacks. 87. For several years, per capita expenditure on prescription drugs in Voronia rose by fifteen percent or more annually. In order to curb these dramatic increases, the ministry of health prohibited drug manufacturers from raising any of their products prices. Even though use of prescription drugs did not expand after this price freeze, per capita expenditure for prescription drugs continued to increase by a substantial percentage each year. Which of the following, if true, most helps to explain why the ministrys action did not achieve its goal?

A. After price increases were prohibited, drug manufacturers concentrated on producing new medications to replace existing products B. The population of Voronia rose steadily throughout the period. C. Improvements in manufacturing processes enable drug manufacturers to maintain high profit levels on drugs despite the price freeze. D. In addition to imposing a price freeze, the government encouraged doctors to prescribe generic versions of common drugs instead of the more expensive brand-name versions E. After price increases were prohibited, some foreign manufacturers of expensive drugs ceased marketing them in Voronia. 88. A cost-effective solution to the problem of airport congestion is to provide highspeed ground transportation between major cities lying 200 to 500 miles apart. The successful implementation of this plan would cost far less than expanding existing airports and would also reduce the number of airplanes clogging both airports and airways. Which of the following, if true, could be proponents of the plan above most appropriately cite as a piece of evidence for the soundness of their plan? A. An effective high-speed ground-transportation system would require major repairs to many highways and mass-transit improvements. B. One-half of all departing flights in the nation's busiest airport head for a destination in a major city 225 miles away. C. The majority of travellers departing from rural airports are flying to destinations in cities over 600 miles away. D. Many new airports are being built in areas that are presently served by high-speed ground-transportation systems. E. A large proportion of air travellers are vacationers who are taking long-distance flights. 89. If there is an oil-supply disruption resulting in higher international oil prices, domestic oil prices in open-market countries such as the United States will rise as well, whether such countries import all or none of their oil. If the statement in the passage concerning oil-supply disruptions is true, which of the following policies in an openmarket nation is most likely to reduce the long-term economic impact on that nation of sharp and unexpected increases in international oil prices? A. Maintaining the quantity of oil imported at constant yearly levels B. Increasing the number of oil tankers in its fleet C. Suspending diplomatic relations with major oil-producing nations D. Decreasing oil consumption through conservation E. Decreasing domestic production of oil 90. The average normal infant born in India weighs between twelve and fourteen pounds at the age of three months. Therefore, if a three-month-old child weighs only ten pounds, its weight gain has been below India's average. Which of the following indicates a flaw in the reasoning above? A. Weight is only one measure of normal infant development. B. Some three-month-old children weigh as much as seventeen pounds. C. It is possible for a normal child to weigh ten pounds at birth. D. The phrase "below average" does not necessarily mean insufficient.

E. Average weight gain is not the same as average weight. 91. Red blood cells in which the malarial-fever parasite resides are eliminated from a person's body after 120 days. Because the parasite cannot travel to a new generation of red blood cells, any fever that develops in a person more than 120 days after that person has moved to a malaria-free region is not due to the malarial parasite. Which is the following, if true, most seriously weakens the conclusion above? A. The fever caused by the malarial parasite may resemble the fever caused by flu viruses. B. The anopheles mosquito, which is the principal insect carrier of the malarial parasite, has been eradicated in many parts of the world. C. Many malarial symptoms other than the fever, which can be suppressed with antimalarial medication, can reappear within 120 days after the medication is discontinued. D. In some cases, the parasite that causes malarial fever travels to cells of the spleen, which are less frequently eliminated from a person's body than are red blood cells. E. In any region infested with malaria-carrying mosquitoes, there are individuals who appear to be immune to malaria. 92. Fact 1: Television advertising is becoming less effective: the proportion of brand names promoted on television that viewers of the advertising can recall is slowly decreasing. Fact 2: Television viewers recall commercials aired first or last in a cluster of consecutive commercials far better than they recall commercials aired somewhere in the middle. Fact 2 would be most likely to contribute to an explanation of fact 1 if which of the following were also true? A. The average television viewer currently recalls fewer than half the brand names promoted in commercials he or she saw. B. The total time allotted to the average cluster of consecutive television commercials is decreasing. C. The average number of hours per day that people spend watching television is decreasing. D. The average number of clusters of consecutive commercials per hour of television is increasing. E. The average number of television commercials in a cluster of consecutive commercials is increasing. 93. Consumer are not so easily manipulated as they are often painted. They may know what they want, and what they want may be greatly different from what other people believe they need. Which of the following statements, if true, most weakens the above argument? a) Most people continue to buy the same brand of a product year after year. b) Companies that advertise the most sell the most products. c) Store shelves packed with a variety of different brands have the potential to confuse the consumer. d) Most consumer know which brand they are going to buy before entering a store. e) Both (c) & (d).

94. The last census showed a sharp rise during the 1970s in the number of Americans living together as unmarried couples, but a more recent increase in the marriage rate in 1981 suggests that matrimony will make a comeback in the 1980s. Which of the following best refutes the argument above? a) One of the causes of more marriages is that the large population resulting from the baby boom is now reaching marriageable age. b) Although information about the 1981 marriage rate is not complete, most analysis considers it to be reliable. c) Many of those marrying in 1981 were couples who had lived together during the 1970s. d) 15/A growing number of private and public initiatives are aimed at strengthening married parent families. e)Here is the truest picture- the marriage rate increased dramatically in 1971 and fell even more dramatically in the following years. 95. Unless new reserves are found soon, the worlds supply of coal is being depleted in such a way that with demand continuing to grow at present rates, reserves will be exhausted by the year 2040. Which of the following, if true, will most weaken the above argument? a)There has been a slowdown in the rate of increase in worlds demand for coal over the last five years from 10% to 5%.784 b)It has been known for many years that there are stocks of coal under Antarctica which have yet to be economically exploited. c)Oil is being used increasingly in place of coal for many industrial & domestic uses. d)Both (a) & (c) e)None of these 96. One way of reducing commuting time for those who work in the cities is to increase the speed at which traffic moves in the heart of the city. This can be accomplished by raising the tolls on the tunnels and bridges connecting the city with other communities. This will discourage auto traffic into the city and will encourage people to use public transportation instead. Which of the following, if true, would LEAST weaken the above argument? a) Nearly all of the traffic in the centre of the city is commercial traffic, which will continue despite toll increases. b) Some people now driving alone into the city would choose to carpool with each other rather than use the public transportation. c) Any temporary improvement in traffic flow would be lost because the improvement itself would attract more cars. d) The numbers of commuters who would be deterred by the toll increases would be insignificant. e) Raising the tolls will not help until the public transport system is improved.

97. From Cochin to Shimla, the new culture vultures are tearing down acres of India'sarchitectural treasures. Ancestral owners are often fobbed off with a few hundred rupeesfor an exquisitely carved door or window, which fetches fifty times that much fromforeign dealers, and yet more from the drawing room sophisticates of Europe and the US.The reason for such shameless rape of the Indian architectural wealth can perhaps notwrongly be attributed to the unfortunate blend of activist disunity and the localindifference.It can be inferred from the above passage that 1. the environment created by the meeting between activist disunity and localindifference is ideal for antique dealers to strive in India. 2. only Indians are not proud of their cultural heritage and are hungry for the foreigncurrency that is easily available in return of artifacts. 3. most Indian families have heirlooms which can be sold at high prices to Europeans andAmericans. 4. India provides a rich market for unscrupulous antique dealers. 98. As part of our program to halt the influx of illegal immigrants, the administration is proposing the creation of a national identity card. The card would be available only to U.S. citizens and to registered aliens, and all persons would be required to produce the card before they could be given a job. Of course, such a system holds the potential, however slight, for the abuse of civil liberties. Therefore, all personal information gathered through this system would be held strictly confidential, to be released only by authorized personnel under appropriate circumstances. Those who are in compliance with U.S. laws would have nothing to fear from the identity card system. In evaluating the above proposal, a person concerned about the misuse of confidential information would be most interested in having the author clarify the meaning of which of the following phrases? (A) "all persons" (line 5) (B) "however slight" (line 7) (C) "civil liberties" (D) "appropriate circumstances" (line 11) (E) "U.S. laws" (line 2) 99. At one time, European and Japanese companies tried to imitate their American rivals. Today, American appliance manufacturers import European scientists to lead their research staffs; American automakers design cars that mimic the styling of German, Italian, and French imports; and American electronics firms boast in their advertising of "Japanese-style" devotion to quality and reliability. In the world of high technology, America has lost the battle for international prestige. Each of the following statements, if true, would help to support the claim above EXCEPT: (A) An American camera company claims in its promotional literature to produce cameras "as fine as the best Swiss imports." (B) An American maker of stereo components designs its products to resemble those of a popular Japanese firm. (C) An American manufacturer of video games uses a brand name chosen because it sounds like a Japanese word. (D) An American maker of televisions studies German-made televisions in order to adopt German manufacturing techniques. (E) An American maker of frozen foods advertises its dinners as "Real European-style entrees prepared by fine French and Italian chefs." 100. Johnson is on firm ground when he asserts that the early editors of ****inson's

poetry often distorted her intentions. Yet Johnson's own, more faithful, text is still guilty of its own forms of distortion. To standardize ****inson's often indecipherable handwritten punctuation by the use of the dash is to render permanent a casual mode of poetic phrasing that ****inson surely never expected to see in print. It implies that ****inson chose the dash as her typical mark of punctuation when, in fact, she apparently never made any definitive choice at all. Which of the following best summarizes the author's main point? (A) Although Johnson is right in criticizing ****inson's early editors for their distortion of her work, his own text is guilty of equally serious distortions. (B) Johnson's use of the dash in his text of ****inson's poetry misleads readers about the poet's intentions. (C) Because ****inson never expected her poetry to be published, virtually any attempt at editing it must run counter to her intentions. (D) Although Johnson's attempt to produce a more faithful text of ****inson's poetry is well-meaning, his study of the material lacks sufficient thoroughness (E) ****inson's editors, including Johnson, have failed to deal adequately with the problem of deciphering ****inson's handwritten manuscripts. 101. A law requiring companies to offer employees unpaid time off to care for their children will harm the economic competitiveness of our nation's businesses. Companies must be free to set their own employment policies without mandated parental-leave regulations. Which of the following, if true, would most seriously weaken the conclusion of the argument above? (A) A parental-leave law will serve to strengthen the family as a social institution in this country. (B) Many businesses in this country already offer employees some form of parental leave. (C) Some of the countries with the most economically competitive businesses have strong parental-leave regulations. (D) Only companies with one hundred or more employees would be subject to the proposed parental-leave law. (E) In most polls, a majority of citizens say they favor passage of a parental-leave law. 102. If A, then B. If B, then C. If C, then D. If all of the statements above are true, which of the following must also be true? (A) If D, then A. (B) If not B, then not C. (C) If not D, then not A. (D) If D, then E (E) If not A, then not D. 103. Dear Applicant: Thank you for your application. Unfortunately, we are unable to offer you a position in our local government office for the summer. As you know, funding for summer jobs is limited, and it is impossible for us to offer jobs to all those who want them. Consequently, we are forced to reject many highly qualified applicants. Which of the following can be inferred from the letter? (A) The number of applicants for summer jobs in the government office exceeded the number of summer jobs available. (B) The applicant who received the letter was considered highly qualified.

(C) Very little funding was available for summer jobs in the government office. (D) The application of the person who received the letter was considered carefully before being rejected (E) Most of those who applied for summer jobs were considered qualified for the available positions. 104. A major film studio announced the release date of a movie based on a novel that, though it was a bestseller when first published, has been out of print for nearly fifteen years. Hoping to capitalize on the anticipated success of the film, the publisher who owns the copyright on the novel plans to print a new edition to be made available the same week the film premieres. EACH of the following, if true, supports the soundness of the publishers plan to capitalize on the success of the film EXCEPT A. The publisher has received permission from the film studio to stamp the words Now a major motion picture on the cover of each book. B. Last year a new edition of a novel that had been out of print hit the bestseller lists two weeks after a movie biography of its author was released. C. The publisher received two hundred thousand dollars after selling the production rights to the film studio. D. The actress playing the lead in the film has discussed her love for the novel in nationally televised interviews. E. Last year a new edition of an unpopular novel was adapted into a top-grossing film, and sales of the book spiked. 105. The price the government pays for standard weapons purchased from military contractors is determined by a pricing method called historical costing. Historical costing allows contractors to protect their profits by adding a percentage increase, based on the current rate of inflation, to the previous years contractual price. Which of the following statements, if true, is the best basis for a criticism of historical costing as an economically sound pricing method for military contracts? A. The government might continue to pay for past inefficient use of funds. B. The rate of inflation has varied considerably over the past twenty years. C. The contractual price will be greatly affected by the cost of materials used for the products. D. Many taxpayers question the amount of money the government spends on military contracts. E. The pricing method based on historical costing might not encourage the development of innovative weapons. 106. Some who favor putting governmental enterprises into private hands suggest that conservation objectives would in general be better served if private environmental groups were put in charge of operating and financing the national park system, which is now run by the government. Which of the following, assuming that it is a realistic possibility, argues most strongly against the suggestion above? A. Those seeking to abolish all restrictions on exploiting the natural resources of the

parks might join the private environmental groups as members and eventually take over their leadership. B. Private environmental groups might not always agree on the best ways to achieve conservation objectives. C. If they wished to extend the park system, the private environmental groups might have to seek contributions from major donors and general public. D. There might be competition among private environmental groups for control of certain park areas. E. Some endangered species, such as the California condor, might die out despite the best efforts of the private environmental groups, even if those groups are not hampered by insufficient resources. 107. A recent spate of launching and operating mishaps with television satellites led to a corresponding surge in claims against companies underwriting satellite insurance. As a result, insurance premiums shot up, making satellites more expensive to launch and operate. This, in turn, has added to the pressure to squeeze more performance out of currently operating satellites. Which of the following, if true, taken together with the information above, best supports the conclusion that the cost of television satellites will continue to increase? A. Since the risk to insurers of satellites is spread over relatively few units, insurance premiums are necessarily very high. B. When satellites reach orbit and then fail, the causes of failure are generally impossible to pinpoint with confidence. C. The greater the performance demands placed on satellites, the more frequently those satellites break down. D. Most satellites are produced in such small numbers that no economies of scale can be realized. E. Since many satellites are built by unwieldy international consortia, inefficiencies are inevitable. 108. Rural households have more purchasing power than do urban or suburban households at the same income level, since some of the income urban and suburban households use for food and shelter can be used by rural households for other needs. Which of the following inferences is best supported by the statement made above? A. The average rural household includes more people than does the average urban or suburban household. B. Rural households have lower food and housing costs than do either urban or suburban households. C. Suburban households generally have more purchasing power than do either rural or urban households. D. The median income of urban and suburban households is generally higher than that of rural households. E. All three types of households spend more of their income on food and housing than on all other purchases combined. 109. In 1985 state border colleges in Bihar lost the enrolment of more than half, on average, of the Nepali nationals they had previously served each year. Teaching faculties

have alleged that this extreme drop resulted from a rise in tuition for international and out-of-state students from Rs 40 to Rs 120 per credit hour. Which of the following, if feasible, offers the best prospects for alleviating the problem of the drop in enrolment of Nepali nationals as the teaching faculties assessed it? A. Providing grants-in-aid to Nepali nationals to study in Nepali universities. B. Allowing Nepali nationals to study in Bihar border colleges and to pay in-state tuition rates, which are the same as the previous international rate C. Reemphasizing the goals and mission of the Bihar state border colleges as serving both in-state students and Nepali nationals D. Increasing the financial resources of Bihar colleges by raising the tuition for in-state students attending state institutions E. Offering career counselling for those Nepali nationals who graduate from state border colleges and intend to return to Nepal 110. Efficiency is all right in its place, in the shop, the factory, the store. The trouble with efficiency is that it wants to rule our play as well as our work; it won't be content to reignin the shop, it follows us home. It can be inferred from the above passage that 1. Efficiency can become all-pervading. 2. Efficiency does not always pay. 3. Efficiency can be more of a torture than a blessing. 4. None of these 111. A major film studio announced the release date of a movie based on a novel that, though it was a bestseller when first published, has been out of print for nearly fifteen years. Hoping to capitalize on the anticipated success of the film, the publisher who owns the copyright on the novel plans to print a new edition to be made available the same week the film premieres. EACH of the following, if true, supports the soundness of the publishers plan to capitalize on the success of the film EXCEPT A. The publisher has received permission from the film studio to stamp the words Now a major motion picture on the cover of each book. B. Last year a new edition of a novel that had been out of print hit the bestseller lists two weeks after a movie biography of its author was released. C. The publisher received two hundred thousand dollars after selling the production rights to the film studio. D. The actress playing the lead in the film has discussed her love for the novel in nationally televised interviews. E. Last year a new edition of an unpopular novel was adapted into a top-grossing film, and sales of the book spiked.

112. Affirmative action is good business. So asserted the National Association of Manufacturers while urging retention of an executive order requiring some federal contractors to set numerical goals for hiring minorities and women. "Diversity in work force participation has produced new ideas in management, product development, and marketing," the association claimed. The association's argument as it is presented in the passage above would be most strengthened if which of the following were true? A. The percentage of minority and women workers in business has increased more slowly than many minority and women's groups would prefer. B. Those businesses with the highest percentages of minority and women workers are those that have been the most innovative and profitable. C. Disposable income has been rising as fast among minorities and women as among the population as a whole. D. The biggest growth in sales in the manufacturing sector has come in industries that market the most innovative products. E. Recent improvements in management practices have allowed many manufacturers to experience enormous gains in worker productivity. 113. If the airspace around centrally located airports were restricted to commercial airliners and only those private planes equipped with radar, most of the private-plane traffic would be forced to sue outlying airfields. Such a reduction in the amount of private-plane traffic would reduce the risk of midair collision around the centrally located airports. The conclusion draw in the first sentence depends on which of the following assumptions? A. Outlying airfields would be as convenient as centrally located airports for most pilots of private planes. B. Most outlying airfields are not equipped to handle commercial-airline traffic. C. Most private planes that use centrally located airports are not equipped with radar. D. Commercial airliners are at greater risk of becoming involved in midair collisions than are private planes. E. A reduction in the risk of midair collision would eventually lead to increases in commercial-airline traffic. 114. If the airspace around centrally located airports were restricted to commercial airliners and only those private planes equipped with radar, most of the private-plane traffic would be forced to sue outlying airfields. Such a reduction in the amount of private-plane traffic would reduce the risk of midair collision around the centrally located airports. Which of the following, if true, would most strengthen the conclusion drawn in the second sentence? A. Commercial airliners are already required by law to be equipped with extremely sophisticated radar systems. B. Centrally located airports are experiencing overcrowded airspace primarily because f sharp increases in commercial-airline traffic. C. Many pilots of private planes would rather buy radar equipment than be excluded from centrally located airports. D. The number of midair collisions that occur near centrally located airports has

decreased in recent years. E. Private planes not equipped with radar systems cause a disproportionately large number of midair collisions around centrally located airports. 115. The overall rate of TB has declined 15% over the last 15 years in Bihar. During that period, the total cost of care for TB sufferers in Bihar, after accounting for inflation, declined by 2% per year until 8 years ago, at which time it began increasing by approximately 2% per year so that now the total health care cost for treating TB is approximately equal to what it was 15 years ago. Which one of the following best resolves the apparent discrepancy between the incidence of TB in Bihar and the cost of caring for TB sufferers? (1) The overall cost of health care in Bihar has increased by 7% in the last 15 years, after accounting for inflation. (2) About 7 years ago, the widespread switch to health maintenance organisations halted overall increases in health care costs in Bihar, after accounting for inflation. (3) Improvements in technology have significantly increased both the costs per patient and the success rate of TB care in the past 15 years. (4) None of the above. 116. Plato said, "A state arises, as I conceive, out of the needs of mankind; no one is self-sufficing, but all of us have many wants". His statement is contradicted by one of the statements given below. Identify that statement. (1) The realization that man needs more than he can fend for himself marks the need for a state. (2) State has nothing to do with self-sufficiency; the first is ridiculously unnecessary and the second is positively achievable. (3) A good state provides equally for all its subjects. (4) The concept of state is not redundant even if man can do away with some surplus needs. 117. In Asia, where palm trees are non-native, the trees' flowers have traditionally been pollinated by hand, which has kept palm fruit productivity unnaturally low. When weevils known to be efficient pollinators of palm flowers were introduced into Asia in 1980, palm fruit productivity increased-by up to fifty percent in some areas-but then decreased sharply in 1984. Which of the following statements, if true, would best explain the 1984 decrease in productivity? A. Prices for palm fruit fell between 1980 and 1984 following the rise in production and a concurrent fall in demand. B. Imported trees are often more productive than native trees because the imported

ones have left behind their pests and diseases in their native lands. C. Rapid increases in productivity tend to deplete trees of nutrients needed for the development of the fruit-producing female flowers. D. The weevil population in Asia remained at approximately the same level between 1980 and 1984. E. Prior to 1980 another species of insect pollinated the Asian palm trees, but not as efficiently as the species of weevil that was introduced in 1980. 118. Since the mayor's publicity campaign for Greenville's bus service began six months ago, morning automobile traffic into the midtown area of the city has decreased seven percent. During the same period, there has been an equivalent rise in the number of persons riding buses into the midtown area. Obviously, the mayor's publicity campaign has convinced many people to leave their cars at home and ride the bus to work. Which of the following, if true, casts the most serious doubt on the conclusion drawn above? A. Fares for all bus routes in Greenville have risen an average of five percent during the past six months. B. The mayor of Greenville rides the bus to City Hall in the city's midtown area. C. Road reconstruction has greatly reduced the number of lanes available to commuters in major streets leading to the midtown area during the past six months. D. The number of buses entering the midtown area of Greenville during the morning hours is exactly the same now as it was one year ago. E. Surveys show that longtime bus riders are no more satisfied with the Greenville bus service than they were before the mayor's publicity campaign began. 119. In the aftermath of a worldwide stock-market crash, Country T claimed that the severity of the stock-market crash it experienced resulted from the accelerated process of denationalization many of its industries underwent shortly before the crash. Which of the following, if it could be carried out, would be most useful in an evaluation of Country T's assessment of the causes of the severity of its stock-market crash? A. calculating the average loss experienced by individual traders in Country T during the crash B. using economic theory to predict the most likely date of the next crash in Country T C. comparing the total number of shares sold during the worst days of the crash in Country T to the total number of shares sold in Country T just prior to the crash

D. comparing the severity of the crash in Country T to the severity of the crash in countries otherwise economically similar to Country T that have not experienced recent denationalization E. comparing the long-term effects of the crash on the purchasing power of the currency of Country T to the immediate, more severe short-term effects of the crash on the purchasing power of the currency of Country T 120. With the emergence of biotechnology companies, it was feared that they would impose silence about proprietary results on their in-house researchers and their academic consultants. This constraint, in turn, would slow the development of biological science and engineering. Which of the following, if true, would tend to weaken most seriously the prediction of scientific secrecy described above? A. Biotechnological research funded by industry has reached some conclusions that are of major scientific importance. B. When the results of scientific research are kept secret, independent researchers are unable to build on those results. C. Since the research priorities of biotechnology companies are not the same as those of academic institutions, the financial support of research by such companies distorts the research agenda. D. To enhance the companies' standing in the scientific community, the biotechnology companies encourage employees to publish their results, especially results that are important. E. Biotechnology companies devote some of their research resources to problems that are of fundamental scientific importance and that are not expected to produce immediate practical applications. 121. Some people have questioned the judge's objectivity in cases of sex discrimination against women. But the record shows that in sixty percent of such cases, the judge has decided in favor of the women. This record demonstrates that the judge has not discriminated against women in cases of sex discrimination against women. The argument above is flawed in that it ignores the possibility that A. a large number of the judge's cases arose out of allegations of sex discrimination against women B. many judges find it difficult to be objective in cases of sex discrimination against women C. the judge is biased against women defendants or plaintiffs in cases that do not involve sex discrimination D. the majority of the cases of sex discrimination against women that have reached the judge's court have been appealed from a lower court E. the evidence shows that the women should have won in more than sixty percent of the judge's cases involving sex discrimination against women 122. The tobacco industry is still profitable and projections are that it will remain so. In the United States this year, the total amount of tobacco sold by tobacco-farmers has

increased, even though the number of adults who smoke has decreased. Each of the following, if true, could explain the simultaneous increase in tobacco sales and decrease in the number of adults who smoke EXCEPT: A. During this year, the number of women who have begun to smoke is greater than the number of men who have quit smoking B. The number of teen-age children who have begun to smoke this year is greater than the number of adults who have quit smoking during the same period C. During this year, the number of non-smokers who have begun to use chewing tobacco or snuff is greater than the number of people who have quit smoking D. The people who have continued to smoke consume more tobacco per person than they did in the past E. More of the cigarettes made in the United States this year were exported to other countries than was the case last year. 123. Kale has more nutritional value than spinach. But since collard greens have more nutritional value than lettuce, if follows that kale has more nutritional value than lettuce. Any of the following, if introduced into the argument as an additional premise, makes the argument above logically correct EXCEPT: A. Collard greens have more nutritional value than kale B. Spinach has more nutritional value than lettuce C. Spinach has more nutritional value than collard greens D. Spinach and collard greens have the same nutritional value E. Kale and collard greens have the same nutritional value 124. On the basis of a decrease in the college-age population, many colleges now anticipate increasingly smaller freshman classes each year. Surprised by a 40 percent increase in qualified applicants over the previous year, however, administrators at Nice College now plan to hire more faculties for courses taken by all freshmen. Which of the following statements about Nice Colleges current qualified applicants, if true, would strongly suggest that the administrators plan is flawed? A. A substantially higher percentage than usual plan to study for advanced degrees after graduation from college. B. According to their applications, their level of participation in extracurricular activities and varsity sports is unusually high. C. According to their applications, none of them lives in a foreign country. D. A substantially lower percentage than usual rate Nice College as their first choice among the colleges to which they are applying E. A substantially lower percentage than usual list mathematics as their intended major. 125. A researcher discovered that people who have low levels of immune-system activity tend to score much lower on tests of mental health than do people with normal or high immune-system activity. The researcher concluded from this experiment that the immune system protects against mental illness as well as against physical disease. The researchers conclusion depends on which of the following assumptions? A. High immune-system activity protects against mental illness better than normal

immune-system activity does. B. Mental illness is similar to physical disease in its effects on body systems. C. People with high immune-system activity cannot develop mental illness. D. Mental illness does not cause peoples immune-system activity to decrease. E. Psychological treatment of mental illness is not as effective as is medical treatment. 126. The effect produced on the mind by travelling depends entirely on the mind of the traveller and on the way in which he conducts himself. The chief idea of one very common type of traveller is to see as many objects of interest as he possibly can. If he can only after his return home say that he has seen such and such a temple, castle, picture gallery, ormuseum, he is perfectly satisfied. Far different is the effect of travels upon those who leave their country with mind prepared by culture to feel intelligent admiration for all the beauties of nature and art to be found in foreign lands. When they visit a new place, instead of hurrying from temple to museum to picture gallery, they allow the spirit of the place to sink into their minds, and only visit such monuments as the time they have at their disposal allows them to contemplate without irreverent haste. It can be inferred from the above passage that 1. the writer prefers the second type of traveller. 2. the first type of traveller is the lay traveller who does not understand the worth of anyplace he travels to. 3. the objective of the second type of traveller is not to see much, but to see well. 4. All of the above 127. The problem of traffic congestion in Athens has been testing the ingenuity of politicians and town planners for years. But the measures adapted to date have not succeeded in decreasing the number of cars on the road in the city centre. In 1980, odds and evens number-plate legislation was introduced under which odd and even plates were banned in the city centre on alternate days thereby expecting to halve the number of cars in the city centre. Then in 1993 it was decreed that all cars in use in the city centre must be fitted with catalytic converters, the only condition being that the buyer of such a 'clean car offered for destruction, a car at least 15-years old. Which one of the following options if true would best support the claim that the measures adopted to date have not succeeded? 1. In the 1980s, many families purchased second cars with the requisite odd or even number plate. 2. In the mid-1990s, many families found it feasible to become first-time car owners by buying a car more than 15 years old and turning it in for a new car with catalytic converters. 3. Post-1993, many families found it feasible to become first-time car owners by buying a car more than 15-years old and buy 'clean' cars from the open market, even if it meant for going the import tax subsidy. 4. All of the above 128. When 100 people who have not used cocaine are tested for cocaine use, on average only 5 will test positive. By contrast, of every 100 people who have used cocaine 99 will test positive. Thus, when a randomly chosen group of people is tested for cocaine use, the vast majority of those who test positive will be people who have used cocaine.

A reasoning error in the argument is that the argument (A) attempts to infer a value judgment from purely factual premises. (B) Attributes to every member of the population the properties of the average member of the population. (C) Fails to take into account what proportion of the population have used cocaine. (D) Ignores the fact that some cocaine users do not test positive. (E) Advocates testing people for cocaine use when there is no reason to suspect that they have used cocaine. 129. With the passage of the new tax reform law the annual tax burden on low-income taxpayers will be reduced, on average, by anywhere from $100 to $300. Clearly, tax reform is in the interest of low-income taxpayers. Which one of the following, if true, most undermines the conclusion above? (A) Tax reform, by simplifying the tax code will save many people the expense of having an accountant do their taxes. (B) Tax reform, by eliminating tax incentives to build rental housing, will push up rents an average of about $40 per month for low-income taxpayers. (C) Low-income taxpayers have consistently voted for those political candidates who are strong advocates of tax reform. (D) The new tax reform laws will permit low and middle-income taxpayers to deduct Child-care expenses from their taxes. (E) Under the new tax reform laws, many low-income taxpayers who now pay taxes will no Longer be required to do so. 130. If we are to expand the exploration of our solar system our next manned flight should be to Phobos, one of Mars's moons, rather than to Mars itself. The flight times to each are the same but the Phobos expedition would require less than half the fuel load of a Mars expedition and would, therefore, be much less costly. So, it is clear that Phobos should be our next step in space exploration. Which one of the following, if true, would most help to explain the difference in fuel requirement? (A) More equipment would be required to explore Phobos than to explore Mars. (B) Smaller spaceships require less fuel than larger spaceships. (C) Information learned during the trip to Phobos can be used during a subsequent trip to Mars. (D) The shortest distance between Phobos and Mars is less than half the shortest distance Between Earth and Mars. (E) Lift-off for the return trip from Phobos requires much less fuel than that from Mars because of Phobos weaker gravitational pull. 131. Using armored vehicles to detonate buried land mines entails an unavoidable risk of injury or fatality, but disarming and removing land mines manually currently entails an even greater such risk to those who remove land mines per mine removed. Therefore, in order to reduce the risk of injury or fatality without decelerating the effort to remove buried land mines, we must increase the use of armored vehicles and disarm fewer land

mines by hand. Which of the following, if true, most seriously undermines the argument above? 1. Manual mine disarmers can be quickly trained in methods that significantly decrease their risk of injury or fatality. 2. Injuries caused by manual disarmament tend to be far more serious than injuries caused by armored vehicle detonations. 3. The delivery of armored vehicles with which to detonate buried land mines can be organized easily by military field operatives. 4. Land mines detonated by armored vehicles destabilize the land surrounding the mine, while land mines that are successfully disarmed manually cause no such damage. 5. Hiring and training those who remove mines by hand is far less costly than is importing heavy armored vehicles to detonate buried land mines. 132. Stock market analyst Dhirubhai Mehta: "We believe that company's stock will appreciate at 35% a year for the next 10-12 years. The company just became the leader and we expect its sales to grow at 8% a year over this period." Investor:"But how can the stock's price be expected to grow more quickly than the company's underlying sales?" Which of the following facts would best support the stock analyst? a. The company's expenses will be declining over the next 5 to 10 years. b. The company just won a patent on a new product. c. Company A's stock is currently overvalued by a significant amount. d. The company's industry peer group is expected to experience stock appreciation rates of 30% over the same time horizon. e. The company is expecting some losses in the coming season. 133. PM Manmohan Singh lobbied for the inclusion of India amongst the set of ASEAN countries. This would help develop and liberalize trade with countries such as Japan,China and other prominent members of the ASEAN group. Each of the following,if true, could account for the above,except a. The PM is up for re-election and needs to show results. b. The PM's United Party Alliance has promised the creation of new jobs in the economy. c. The inclusion of India in the ASEAN would be a major achievement on the economic and political front for Manmohan Singh. d. Being a shrewd economist, Manmohan Singh realised that trade agreements with prominent nations of the ASEAN woul definitely lead to an increase in the trade deficit of the country. e. The IMF would punish the PM for such a deal.

134. A car magazine report: 'The average mileage in the small car market was found to be 18 kilometers per litre. The average mileage was calculated by taking cars of all manufactures in the segment, filling them with 10 litres of fuel and driving them along the Mumbai-Pune expressway. However, for the Karuti, the mileage was 22 kilometers per litre. Clearly,if you want to buy a new car, you should buy the Karuti.' Which of the following additional information makes the argument stronger? I.Petrol prices have touched the sky, and hence the reader should be primarily interested in saving on fuel costs while purchasing a new car. II. Besides mileage, Karuti scored the best on 9 out of 10 performance indicators used by the survey. III. The article is titled-"Your first Car". a. I and II b. II and III c. II only d. All of these e. None of these. 135. Scientific research that involves international collaboration has produced papers of greater influence, as measured by the number of times a paper is cited in subsequent papers, than has research without any collaboration. Papers that result from international collaboration are cited an average of seven times, whereas papers with single authors are cited only three times on average. This difference shows that research projects conducted by international research teams are of greater importance than those conducted by single researchers. Which one of the following is an assumption on which the argument depends? (A) Prolific writers can inflate the number of citations they receive by citing themselves in subsequent papers. (B) It is possible to ascertain whether or not a paper is the product of international collaboration by determining the number of citations it has received (C) The number of citations a paper receives is a measure of the importance of the research it reports. (D) The collaborative efforts of scientists who are citizens of the same country do not produce papers that are as important as papers that are produced by international collaboration. (E) International research teams tend to be more generously funded than are single researchers. 136. It is more desirable to have some form of socialized medicine than a system of medical care relying on the private sector. Socialized medicine is more broadly accessible than is private-sector system. In addition, since countries with socialized medicine have a lower infant mortality rate than do countries with a system relying entirely on the private sector, socialized medicine seems to be technologically superior. Which one of the following best indicates a flaw in the argument about the technological superiority of socialized medicine?

(A) The lower infant mortality rate might be due to the systems allowing greater access to Medical care (B) There is no necessary connection between the economic system of socialism and Technological achievement. (C) Infant mortality is a reliable indicator of the quality of medical care for children. (D) No list is presented of the countries whose infant mortality statistics are summarized under the two categories, socialized and private-sector. (E) The argument presupposes the desirability of socialized medicine, which is what the Argument seeks to-establish. 137. Lourdes: Dietary fibre is an important part of a healthful diet. Experts recommend that adults consume 20 to 35 grams of fibre a day. Kyra: But a daily intake of fibre that is significantly above that recommended level interferes with mineral absorption, especially the absorption of calcium. The public should be told to cut act on fibre intake Which one of the following, if true, most undermines Kyra's recommendation? (A) Among adults, the average consumption of dietary fibre is at present approximately 10 grams a day. (B) The more a food is processed, the more the fibre is broken down and the lower the fibre content. (C) Many foodstuffs that are excellent sources of fibre are economical and readily available (D) Adequate calcium intake helps prevent the decrease in bone mass known as osteoporosis. (E) Many foodstuffs that are excellent sources of fibre are popular with consumers. 138. Scientific research that involves international collaboration has produced papers of greater influence, as measured by the number of times a paper is cited in subsequent papers, than has research without any collaboration. Papers that result from international collaboration are cited an average of seven times, whereas papers with single authors are cited only three times on average. This difference shows that research projects conducted by international research teams are of greater importance than those conducted by single researchers. Which one of the following is an assumption on which the argument depends? (A) Prolific writers can inflate the number of citations they receive by citing themselves in subsequent papers. (B) It is possible to ascertain whether or not a paper is the product of international collaboration by determining the number of citations it has received (C) The number of citations a paper receives is a measure of the importance of the research it reports. (D) The collaborative efforts of scientists who are citizens of the same country do not produce papers that are as important as papers that are produced by international collaboration. (E) International research teams tend to be more generously funded than are single researchers.

139. It is more desirable to have some form of socialized medicine than a system of medical care relying on the private sector. Socialized medicine is more broadly accessible than is private-sector system. In addition, since countries with socialized medicine have a lower infant mortality rate than do countries with a system relying entirely on the private sector, socialized medicine seems to be technologically superior. Which one of the following best indicates a flaw in the argument about the technological superiority of socialized medicine? (A) The lower infant mortality rate might be due to the systems allowing greater access to Medical care (B) There is no necessary connection between the economic system of socialism and Technological achievement. (C) Infant mortality is a reliable indicator of the quality of medical care for children. (D) No list is presented of the countries whose infant mortality statistics are summarized under the two categories, socialized and private-sector. (E) The argument presupposes the desirability of socialized medicine, which is what the Argument seeks to-establish. 140. Lourdes: Dietary fibre is an important part of a healthful diet. Experts recommend that adults consume 20 to 35 grams of fibre a day. Kyra: But a daily intake of fibre that is significantly above that recommended level interferes with mineral absorption, especially the absorption of calcium. The public should be told to cut act on fibre intake Which one of the following, if true, most undermines Kyra's recommendation? (A) Among adults, the average consumption of dietary fibre is at present approximately 10 grams a day. (B) The more a food is processed, the more the fibre is broken down and the lower the fibre content. (C) Many foodstuffs that are excellent sources of fibre are economical and readily available (D) Adequate calcium intake helps prevent the decrease in bone mass known as osteoporosis. (E) Many foodstuffs that are excellent sources of fibre are popular with consumers. 141. Policy Adviser: Freedom of speech is not only a basic human right, it is also the only rational policy for this government to adopt. When ideas are openly aired good idea flourish, silly proposals are easily recognized as such, and dangerous ideas can be responded to by forcing citizens to disseminate their thoughts in secret. Which one of the following, if true, world most strengthen the argument? (A) Most citizens would tolerate some limits on freedom of speech (B) With or without a policy of freedom of speech, governments respond to dangerous ideas irrationally (C) Freedom of religion and freedom of assembly are also basic human rights than governments must recognize (D) Governments are less likely to be overthrown if they openly adopt a policy allowing

freedom of speech (E) Great ideas have flourished in societies that repress free speech as often as in those that permit it 142. Book Review: When I read a novel set in a city I know well, I must see that the writer knows the city as well as I do if I am to take that writer seriously. If the writer is faking I know immediately and do not trust the writer. When a novelist demonstrates the required knowledge, I trust the story teller, so I trust the tale. This trust increases my enjoyment of a good novel. Peter Lee's second novel is set in San Francisco, in this novel, as in his first, Lee passes my test with flying colours. Which one of the following can be properly inferred from the passage? (A) The book reviewer enjoys virtually any novel written by a novelist whom she trusts (B) If the book reviewer trusts the novelist as a storyteller, the novel in question must be set in a city the book reviewer knows well (C) Peter Lee's first novel was set in San Francisco (D) The book reviewer does not trust any novel set in a city that she does not know well (E) The book reviewer does not believe that she knows San Francisco better than Peter Lee does 143. Prominent business executives often play active roles in United States presidential campaigns as fundraisers or backroom strategists but few actually seek to become president themselves. Throughout history the great majority of those who have sought to become president have been lawyers, military leaders, or full-time politicians. This is understandable, for the personality and skills that make for success in business do not make for success in politics. Business is largely hierarchical, whereas politics is coordinative; as a result, business executives tend to be uncomfortable with compromises and power sharing, which are inherent in politics. Which one of the following, if true, most seriously weakens the proposed explanation of why business executives do not run for president? (A) Many of the most active presidential fundraisers and backroom strategists are themselves politicians. (B) Military leaders are generally no more comfortable with compromises and power sharing than are business executives. (C) Some of the skills needed to become a successful lawyer are different from some of those needed to become a successful military leader. (D) Some former presidents have engaged in business ventures after leaving office (E) Some hierarchically structured companies have been major financial supporters of candidates for president. 144. A scientific theory is a good theory if it satisfies two requirements - it must accurately describe a large class of observations in terms of a model that is simple enough to contain only a few elements, and it must make definite predictions about the results of future observations. For example, Aristotles cosmological theory, which claimed that everything was made out of four elements - earth, air, fire, and water - satisfied the first requirement but it did not make any definite prediction. Thus, Aristotle's cosmological theory was not a good theory.

If all the statements in the passage are true, each of the following must also be true EXCEPT: (A) Prediction about the results of future observations must be made by any good scientific theory. (B) Observation of physical phenomena was not a major concern in Aristotle's cosmological Theory (C) Four elements can be the basis of a scientific model that is simple enough to meet the Simplicity criterion of a good theory. (D) A scientific model that contains many elements is not a good theory (E) Aristotles cosmological theory described a large class of observations in terms of only four elements. 145. New satellite imagery and overflights have revealed more than 200 huge geometric symbol carved on the Earth's surface in the upper Amazon basin near Brazil's border with Bolivia. Spanning 250 km, the circles, squares and other geometric shapes form a network of avenues, ditches and enclosures. The discoveries are challenging long held assumptions about the Amazon as a Hobbesian place where only small primitive tribes could ever have existed and about the limits the environment placed on the rise of early civilizations. Which of the following, if true, is an assumption to reach the conclusion as the above one? a) Knowledge of geometry is a feature exhibited by the civilized tribes b) Primitive tribes were barbaric in nature. c) Construction of symmetrical structures is a characteristic exhibited by the civilized people. d) Like other early civilizations, the Amazon river valley must have provided the requirements for the springing up of human civilization. 146. Vasquez-Morrell Assurance specializes in insuring manufacturers. Whenever a policyholder makes a claim, a claims adjuster determines the amount that Vasquez-Morrell is obligated to pay. VasquezMorrell is cutting its staff of claims adjusters by 15 percent. To ensure that the companys ability to handle claims promptly is affected as little as possible by the staff cuts, consultants recommend that Vasquez-Morrell lay off those adjusters who now take longest, on average, to complete work on claims assigned to them. Which of the following, if true, most seriously calls into question the consultants criterion for selecting the staff to be laid off? (A) If the time that Vasquez-Morrell takes to settle claims increases significantly, it could lose business to other insurers. (B) Supervisors at Vasquez-Morrell tend to assign the most complex claims to the most capable adjusters. (C) At Vasquez-Morrell, no insurance payments are made until a claims adjuster has reached a final determination on

the claim. (D) There are no positions at Vasquez-Morrell to which staff currently employed as claims adjusters could be reassigned. (E) The premiums that Vasquez-Morrell currently charges are no higher than those charged for similar coverage by competitors. 147. Organisations are often defined as groups of people who come together to pursue a common goal. But more often than not, goals diverge as much as they converge, making the rationality of the overall organisation no more than an elusive ideal. Beneath the collective irrationality, however, organisations are often operating in a way that is eminently rational from the standpoint of the individuals, groups and coalitions directly involved. Which of the following can be inferred from the above passage? i) If all the employees of an organization pursue their individual goals, one can never have an organization that behaves rationally. ii) Although conceptually an organization may appear to be irrational, behaviours of individuals, groups and coalitions in the organisation may be rational. iii) As individuals, groups or coalitions in an organization pursue their own interests, the conceptual issues of rational behaviour get blurred. iv) Since people are essentially irrational, the ideal of building a rational organisation is elusive. 148. Deepa Mehtas Fire is under fire from the countrys self-appointed moral police. Their contention is that the film is a violation of the Indian cultural moral and cannot be allowed to influence the Indian psyche. According to them such films ruin the moral fabric of the nation which must be protected and defended against such intrusions at all cost even at the cost of cultural dictatorship. Based on the information in the above passage it can be inferred that 1. the assumption underlying the moral polices critique of Fire is that the Indian audience is vulnerable to all types of influence. 2. the assumption underlying the moral polices critique of Fire is that Indian audience is impressionable and must be protected against immoral influences. 3. the moral police thinks it has the sole authority to pass judgment of films screened in India. 4. None of the above 149. Virtually all health experts agree that second-hand smoke poses a serious health risk. After the publication of yet another research paper explicating the link between exposure to second-hand smoke and a shorter life span, some members of the State House of Representatives proposed a ban on smoking in most public places in an attempt to promote quality of life and length of lifespan. Which of the following, if true, provides the most support for the actions of the State Representatives? A) The amount of damaging chemicals and fumes released into the air by cigarette smoke is far less than the amount released from automobiles, especially from older models.

B) Banning smoking in most public places will not considerably reduce the percent of the population in the state in question that smokes. C) The state whose legislators are proposing the tough smoking legislation has a relatively high percent of its population that smoke. D) Another state that enacted a similar law a decade ago saw a statistically significant drop in lung-cancer rates among non-smokers. E) A nearby state up-wind has the highest number of smokers in the country. 150. Developed countries have made adequate provisions for social security for senior citizens. State insurers (as well as private ones) offer medicare and pension benefits to people who can no longer earn. In India, with the collapse of the joint family system, the traditional shelter of the elderly has disappeared. And a State faced with a financial crunch is not in a position to provide social security. So, it is advisable that the working population gives serious thought to build a financial base for itself. Which one of the following, if it were to happen, weakens the conclusion drawn in the above passage the most? The investible income of the working population, as a proportion of its total income, will grow in the future. The insurance sector is underdeveloped and trends indicate that it will be extensively privatised in the future. India is on a path of development that will take it to a developed country status, with all its positive and negative implications. If the working population builds a stronger financial base, there will be a revival of the joint family system. 151. Net Neutrality stipulates that Internet service providers (ISP) cannot partition their bandwidth such that different types of Internet communications have different maximum bandwidth capacities. For example, an ISP cannot relegate high bandwidth voice-over-IP (VoIP) traffic to a separate tunnel in an attempt to ensure that users of low-bandwidth functions such as plain-text email are not slowed down by the high-bandwidth users. Some individuals support implementing Net Neutrality on the principle that one group (i.e., users of high-bandwidth services) should not be effectively penalized for the actions of another group (i.e., users of slow-bandwidth services, who have a special traffic lane carved out for them, thereby slowing high-bandwidth users). Which of the following, if true, most seriously weakens the argument of the supporters of Net Neutrality mentioned above? A) The jobs of many high-bandwidth users require these individuals to use highbandwidth services. B) Placing no restrictions on the bandwidth of individuals who use high-bandwidth services would force ISPs to purchase massive amounts of expensive additional bandwidth, disproportionately increasing the price of access for low-bandwidth users. C) A strong and well respected lobbying firm recently revealed it has been hired by large telecommunications firms to oppose Net Neutrality on the grounds that it infringes upon a private company's ability to do business.

D) One country that mandated Net Neutrality saw a decrease in satisfaction of Internet users. E) A recent court ruling upheld the principle that technology companies cannot discriminate in whom they serve or how they serve users. 152.Eating beets significantly lowers the risk of cancer, according to an article in a nutritional magazine. The article refers to a study that found that people who consumed one or more beets per day were half as likely to be diagnosed with the disease as people who did not. Which of the following, if true, most weakens the argument in the magazine article? A) Another study found that people who consumed one tablespoon of flax seed oil per day were more than four times less likely to be diagnosed with cancer as those who did not. B) Participants in the study reported consuming no vegetables other than beets. C) The study was only conducted in one city. D) In another experiment, cancer patients who ate one or more beets per day were no more likely to recover than those who ate no beets. E) The participants in the study who ate beets were more likely to exercise regularly than those who did not eat beets. 153. A certain automaker aims to increase its market share by deeply discounting its vehicles' prices for the next several months. The discounts will cut into prots, but because they will be heavily advertised the manufacturer hopes that they will attract buyers away from rival manufacturers' cars. In the longer term, the automaker envisions that customers initially attracted by the discounts may become loyal customers. In assessing the plan's chances of achieving its aim, it would be most useful to know which of the following? (A) Whether the automaker's competitors are likely to respond by offering deep discounts on their own products (B) Whether the advertisements will be created by the manufacturer's current advertising agency (C) Whether some of the automaker's models will be more deeply discounted than others (D) Whether the automaker will be able to cut costs suf ciently to maintain pro t margins even when the discounts are in effect (E) Whether an alternative strategy might enable the automaker to enhance its pro tability while holding a constant or diminishing share of the market.

154. In Swartkans territory, archaeologists discovered charred bone fragments dating back one million years. Analysis of the fragments, which came from a variety of animals, showed that they had been heated to temperatures no higher than those produced in experimental campres made from branches of white stinkwood, the most common tree around Swartkans. Which of the following, if true, would, together with the information above, provide the best basis for the claim that the charred bone fragments are evidence of the use of re by early hominids? (A) The white stinkwood tree is used for building material by the present-day inhabitants of Swartkans. (B) Forest res can heat wood to a range of temperatures that occur in campres. (C) The bone fragments were tted together by the archaeologists to form the complete skeletons of several animals. (D) Apart from the Swartkans discovery, there is reliable evidence that early hominids used re as many as 500,000 years ago. (E) The bone fragments were found in several distinct layers of limestone that contained primitive cutting tools known to have been used by early hominids. 155. Male bowerbirds construct elaborately decorated nests, or bowers. Basing their judgment on the fact that different local populations of bowerbirds of the same species build bowers that exhibit different building and decorative styles, researchers have concluded that the bowerbirds' building styles are a culturally acquired, rather than a genetically transmitted, trait. Which of the following, if true, would most strengthen the conclusion drawn by the researchers? (A) There are more common characteristics than there are differences among the bowerbuilding styles of the local bowerbird population that has been studied most extensively. (B) Young male bowerbirds are inept at bower-building and apparently spend years watching their elders before becoming accomplished in the local bower style. (C) The bowers of one species of bowerbird lack the towers and ornamentation characteristic of the bowers of most other species of bowerbird. (D) Bowerbirds are found only in New Guinea and Australia, where local populations of the birds apparently seldom have contact with one another. (E) It is well known that the song dialects of some songbirds are learned rather than

transmitted genetically. 156. A conservation group in the United States is trying to change the long-standing image of bats as frightening creatures. The group contends that bats are feared and persecuted solely because they are shy animals that are active only at night. Which of the following, if true, would cast the most serious doubt on the accuracy of the group's contention? (A) Bats are steadily losing natural roosting places such as caves and hollow trees and are thus turning to more developed areas for roosting. (B) Bats are the chief consumers of nocturnal insects and thus can help make their hunting territory more pleasant for humans. (C) Bats are regarded as frightening creatures not only in the United States but also in Europe, Africa, and South America. (D) Raccoons and owls are shy and active only at night, yet they are not generally feared and persecuted. (E) People know more about the behavior of other greatly feared animal species, such as lions, alligators, and snakes, than they do about the behavior of bats. 157. In recent years, many cabinetmakers have been winning acclaim as artists. But since furniture must be useful, cabinetmakers must exercise their craft with an eye to the practical utility of their product. For this reason, cabinetmaking is not art. Which of the following is an assumption that supports drawing the conclusion above from the reason given for that conclusion? (A) Some furniture is made to be placed in museums, where it will not be used by anyone. (B) Some cabinetmakers are more concerned than others with the practical utility of the products they produce. (C) Cabinetmakers should be more concerned with the practical utility of their products than they currently are. (D) An object is not an art object if its maker pays attention to the object's practical utility. (E) Artists are not concerned with the monetary value of their products. 158. Plan: Concerned about the welfare of its senior citizens, the government of Runagia decided two years ago to increase by 20 percent the government-provided pension paid to all Runagians age sixty- ve and older.

Result: Many Runagian senior citizens are no better off nancially now than they were before the increase. Further information: The annual rate of in ation since the pension increase has been below 5 percent, and the increased pension has been duly received by all eligible Runagians. In light of the further information, which of the following, if true, does most to explain the result that followed implementation of the plan? (A) The majority of senior citizens whose nancial position has not improved rely entirely on the government pension for their income. (B) The Runagian banking system is so inef cient that cashing a pension check can take as much as three weeks. (C) The prices of goods and services that meet the special needs of many senior citizens have increased at a rate much higher than the rate of in ation. (D) The pension increase occurred at a time when the number of Runagians age sixty- ve and older who were living below the poverty level was at an all-time high. (E) The most recent pension increase was only the second such increase in the last ten years. 159. Investigators concluded that human failure was not responsible for the fatal airplane crash last August, and since that time new and more stringent rules for identifying and reporting mechanical problems have been in effect. That accounts for the fact that reports of airplane mechanical problems have increased in frequency by 50 percent since last August. Which one of the following is an assumption underlying the argument in the passage? (A) Airplane travel is still relatively safe, despite the increase in reported mechanical problems. (B) Mechanical problems in airplanes have increased dramatically since last August. (C) Mechanical problems in airplanes have not increased by 50 percent since last August. (D) Airlines are less reluctant to report mechanical problems than they previously were. (E) Mechanical problems in airplanes have become easier to detect since last August 160. Following several years of declining advertising sales,Greenville times reorganized its advertising salesforce two years ago.Before reorgansation,salesforcewas organised geographically,with some reps concentrating on city centre businesses and others on different outlying regions.The reorganisation attempted to increse the sales reps's knowledge of client's businesses by having each rep deal with only one type of industry or of retailing.After this,advertising sales increased. In accessing whether the impovement in sales was due to the reorganisation,each of the the following will be useful except: 1>2 years ago,what proportion of greenville times sales was generated by adv sales? 2>haqs the circulation increased substantially in lsat 2 yrs?

3>has there been a substatial turnover in personnel in the advertising slaes force over last 2 yrs? 4>has the econoy in greenville and surrounding areas been growing rapily for past 2 yrs? 161. Since it has become known that several of a bank's top executives have been buying shares in their own bank, the bank's depositors, who had been worried by rumors that the bank faced impending financial collapse, have been greatly relieved. They reason that since top executives evidently have faith in the bank's financial soundness, those worrisome rumors must be false. They might well be overoptimistic, however since corporate executives have sometimes bought shares in their own company in a calculated attempt to dispel negative rumors about the company's health. In the argument given, the two boldfaced portions play which of the following roles? A. The first summarizes the evidence used in the reasoning called into question by the argument; the second states the counterevidence on which the argument relies. B. The first summarizes the evidence used in the reasoning called into question by the argument; the second is an intermediate conclusion supported by the evidence. C. The first is an intermediate conclusion that forms part of the reasoning called into question by the argument; the second is evidence that undermines the support for this intermediate conclusion. D. The first is an intermediate conclusion that forms part of the reasoning called into question by the argument; the second is the main conclusion of the argument. E. The first is an intermediate conclusion that forms part of the reasoning called into question by the argument; the second states a further conclusion supported by this intermediate conclusion. 162. In the past, most children who went sledding in the winter snow in Verland used wooden sleds with runners and steering bars. Ten years ago, smooth plastic sleds became popular; they go faster than wooden sleds but are harder to steer and slow. The concern that plastic sleds are more dangerous is clearly borne out by the fact that the number of children injured while sledding was much higher last winter than it was ten years ago. Which of the following, if true in Verland, most seriously undermines the force of the evidence cited? A. A few children still use traditional wooden sleds. B. Very few children wear any kind of protective gear, such as helmets, while sledding. C. Plastic sleds can be used in a much wider variety of snow conditions than wooden sleds can. D. Most sledding injuries occur when a sled collides with a tree, a rock, or another sled. E. Because the traditional wooden sled can carry more than one rider, an accident involving a wooden sled can result in several children being injured. 163. The spacing of the four holes on a fragment of a bone flute excavated at a Neanderthal campsite is just what is required to play the third through sixth notes of the diatonic scale-the seven-note musical scale used in much of Western music since the Renaissance. Musicologists therefore hypothesize that the diatonic musical scale was developed and used thousands of years before it was adopted by Western musicians. Which of the following, if true, most strongly supports the hypothesis? A. Bone flutes were probably the only musical instrument made by Neanderthals. B. No musical instrument that is known to have used a diatomic scale is of an earlier

date than the flute found at the Neanderthal campsite. C. The flute was made from a cave-bear bone and the campsite at which the flute fragment was excavated was in a cave that also contained skeletal remains of cave bears. D. Flutes are the simplest wind instrument that can be constructed to allow playing a diatonic scale. E. The cave-bear leg bone used to make the Neanderthal flute would have been long enough to make a flute capable of playing a complete diatonic scale 164. For a local government to outlaw all strikes by its workers is a costly mistake, because all its labor disputes must then be settled by binding arbitration, without any negotiated public-sector labor settlements guiding the arbitrators. Strikes should be outlawed only for categories of public-sector workers for whose services no acceptable substitute exists. The statements above best support which of the following conclusions? (A) Where public-service workers are permitted to strike, contract negotiations with those workers are typically settled without a strike. (B) Where strikes by all categories of public-sector workers are outlawed, no acceptable substitutes for the services provided by any of those workers are available. (C) Binding arbitration tends to be more advantageous for public-service workers where it is the only available means of settling labor disputes with such workers. (D) Most categories of public-sector workers have no counterparts in the private sector. (E) A strike by workers in a local government is unlikely to be settled without help from an arbitrator. 165. The growing popularity of computer-based activities was widely expected to result in a decline in television viewing, since it had been assumed that people lack suffi cient free time to maintain current televisionviewing levels while spending increasing amounts of free time on the computer. That assumption, however, is evidently false: In a recent mail survey concerning media use, a very large majority of respondents who report increasing time spent per week using computers report no change in time spent watching television. Which of the following would it be most useful to determine in order to evaluate the argument? (A) Whether a large majority of the survey respondents reported watching television regularly. (B) Whether the amount of time spent watching television is declining among people who report that they rarely or never use computers. (C) Whether the type of television programs a person watches tends to change as the amount of time spent per week using computers increases. (D) Whether a large majority of the computer owners in the survey reported spending increasing amounts of time per week using computers.

(E) Whether the survey respondents' reports of time spent using computers included time spent using computers at work. 166. Crowding on Mooreville's subway frequently leads to delays, because it is diffi cult for passengers to exit from the trains. Subway ridership is projected to increase by 20 percent over the next 10 years. The Mooreville Transit Authority plans to increase the number of daily train trips by only 5 percent over the same period. Offi cials predict that this increase is sufficient to ensure that the incidence of delays due to crowding does not increase. Which of the following, if true, provides the strongest grounds for the officials' prediction? (A) By changing maintenance schedules, the Transit Authority can achieve the 5 percent increase in train trips without purchasing any new subway cars. (B) The Transit Authority also plans a 5 percent increase in the number of bus trips on routes that connect to subways. (C) For most commuters who use the subway system, there is no practical alternative public transportation available. (D) Most of the projected increase in ridership is expected to occur in off-peak hours when trains are now sparsely used. (E) The 5 percent increase in the number of train trips can be achieved without an equal increase in Transit Authority operational costs. 167. Some anthropologists study modern-day societies of foragers in an effort to learn about our ancient ancestors who were also foragers. A flaw in this strategy is that forager societies are extremely varied. Indeed, any forager society with which anthropologists are familiar has had considerable contact with modern, non-forager societies. Which of the following, if true, would most weaken the criticism made above of the anthropologists' strategy? (A) All forager societies throughout history have had a number of important features in common that are absent from other types of societies. (B) Most ancient forager societies either dissolved or made a transition to another way of life. (C) All anthropologists study one kind or another of modern-day society. (D) Many anthropologists who study modern-day forager societies do not draw inferences about ancient societies on the basis of their studies. (E) Even those modern-day forager societies that have not had significant contact with modern societies are importantly different from ancient forager societies.

168. The argument for liberalization which answers the worries of the left parties about the possible trade deficits created by the opening up of the Indian economy goes thus : 'In today's economic scenario,where there are many trading countries, the trade between two specific countries need not be balanced. The differing demands for goods and services and the differing productive capabilities of the same among different countries will cause a country like India to have trade deficits with some countries and surpluses with other countries. On the whole, the trade deficits and surpluses will balance out in order to give a trade balance.' Which of the following conclusions best summarises the argument present in the passage above? a. Left parties need not worry about trade deficits in India since its trade will always be in balance even though it runs a deficit with a single country. b. India's trade deficits and surpluses with other countries always balance out. c. The left parties in India should not be concerned about India's trade deficits with specific countries because they will balance out in the long run. d. None of these e. Only (a) and (b) 169. Red blood cells in which the malarial-fever parasite resides are eliminated from a person's body after 120 days. Because the parasite cannot travel to a new generation of red blood cells, any fever that develops in a person more than 120 days after that person has moved to a malaria-free region is not due to the malarial parasite. Which of the following, if true, most seriously weakens the conclusion above? (A) The fever caused by the malarial parasite may resemble the fever caused by flu viruses. (B) The anopheles mosquito, which is the principal insect carrier of the malarial parasite, has been eradicated in many parts of the world. (C) Many malarial symptoms other than the fever, which can be suppressed with antimalarial medication, can reappear within 120 days after the medication is discontinued. (D) In some cases, the parasite that causes malarial fever travels to cells of the spleen, which are less frequently eliminated from a person's body than are red blood cells. (E) In any region infested with malaria-carrying mosquitoes, there are individuals who appear to be immune to malaria. 170. When there is less rainfall than normal, the water level of Australian rivers falls and the rivers fl ow more slowly. Because algae whose habitat is river water grow best in slow-moving water, the amount of algae per unit of water generally increases when there has been little rain. By contrast, however, following a period of extreme drought, algae levels are low even in very slow-moving river water.

Which of the following, if true, does most to explain the contrast described above? (A) During periods of extreme drought, the populations of some of the species that feed on algae tend to fall. (B) The more slowly water moves, the more conducive its temperature is to the growth of algae. (C) When algae populations reach very high levels, conditions within the river can become toxic for some of the other species that normally live there. (D) Australian rivers dry up completely for short intervals in periods of extreme drought. (E) Except during periods of extreme drought, algae levels tend to be higher in rivers in which the flow has been controlled by damming than in rivers that fl ow freely. 171. When hypnotized subjects are told that they are deaf and are then asked whether they can hear the hypnotist, they reply, "No." Some theorists try to explain this result by arguing that the selves of hypnotized subjects are dissociated into separate parts, and that the part that is deaf is dissociated from the part that replies. Which of the following challenges indicates the most serious weakness in the attempted explanation described above? (A) Why does the part that replies not answer, "Yes"? (B) Why are the observed facts in need of any special explanation? (C) Why do the subjects appear to accept the hypnotist's suggestion that they are deaf? (D) Why do hypnotized subjects all respond the same way in the situation described? (E) Why are the separate parts of the self the same for all subjects? 172. A prominent investor who holds a large stake in the Burton Tool Company has recently claimed that the company is mismanaged, citing as evidence the company's failure to slow production in response to a recent rise in its inventory of finished products. It is doubtful whether an investor's sniping at management can ever be anything other than counterproductive, but in this case it is clearly not justified. It is true that an increased inventory of fi nished products often indicates that production is outstripping demand, but in Burton's case it indicates no such thing. Rather, the increase in inventory is entirely attributable to products that have already been assigned to orders received from customers. In the argument given, the two boldfaced portions play which of the following roles? (A) The fi rst states the position that the argument as a whole opposes; the second provides evidence to undermine the support for the position being opposed. (B) The fi rst states the position that the argument as a whole opposes; the second is evidence that has been used to support the position being opposed. (C) The fi rst states the position that the argument as a whole opposes; the second states the conclusion of the argument as a whole.

(D) The first is evidence that has been used to support a position that the argument as a whole opposes; the second provides information to undermine the force of that evidence. (E) The first is evidence that has been used to support a position that the argument as a whole opposes; the second states the conclusion of the argument as a whole. 173. Several industries have recently switched at least partly from older technologies powered by fossil fuels to new technologies powered by electricity. It is thus evident that less fossil fuel is being used as a result of the operations of these industries than would have been used if these industries had retained their older technologies. Which of the following, if true, most strengthens the argument above? (A) Many of the industries that have switched at least partly to the new technologies have increased their output. (B) Less fossil fuel was used to manufacture the machinery employed in the new technologies than was originally used to manufacture the machinery employed in the older technologies. (C) More electricity is used by those industries that have switched at least partly to the new technologies than by those industries that have not switched. (D) Some of the industries that have switched at least partly to the new technologies still use primarily technologies that are powered by fossil fuels. (E) The amount of fossil fuel used to generate the electricity needed to power the new technologies is less than the amount that would have been used to power the older technologies. 174. A recent report determined that although only 3 percent of drivers on Maryland highways equipped their vehicles with radar detectors, 33 percent of all vehicles ticketed for exceeding the speed limit were equipped with them. Clearly, drivers who equip their vehicles with radar detectors are more likely to exceed the speed limit regularly than are drivers who do not. The conclusion drawn above depends on which of the following assumptions? (A) Drivers who equip their vehicles with radar detectors are less likely to be ticketed for exceeding the speed limit than are drivers who do not. (B) Drivers who are ticketed for exceeding the speed limit are more likely to exceed the speed limit regularly than are drivers who are not ticketed. (C) The number of vehicles that were ticketed for exceeding the speed limit was greater than the number of vehicles that were equipped with radar detectors. (D) Many of the vehicles that were ticketed for exceeding the speed limit were ticketed more than once in the time period covered by the report. (E) Drivers on Maryland highways exceeded the speed limit more often than did drivers on other state highways not covered in the report. 175. In countries where automobile insurance includes compensation for whiplash

injuries sustained in automobile accidents, reports of having suffered such injuries are twice as frequent as they are in countries where whiplash is not covered. Presently, no objective test for whiplash exists, so it is true that spurious reports of whiplash injuries cannot be readily identifi ed. Nevertheless, these facts do not warrant the conclusion drawn by some commentators that in the countries with the higher rates of reported whiplash injuries, half of the reported cases are spurious. Clearly, in countries where automobile insurance does not include compensation for whiplash, people often have little incentive to report whiplash injuries that they actually have suffered. In the argument given, the two boldfaced portions play which of the following roles? (A) The first is a claim that the argument disputes; the second is a conclusion that has been based on that claim. (B) The first is a claim that has been used to support a conclusion that the argument accepts; the second is that conclusion. (C) The first is evidence that has been used to support a conclusion for which the argument provides further evidence; the second is the main conclusion of the argument. (D) The first is a finding whose implications are at issue in the argument; the second is a claim presented in order to argue against deriving certain implications from that finding. (E) The first is a finding whose accuracy is evaluated in the argument; the second is evidence presented to establish that the finding is accurate. 176. Many people suffer an allergic reaction to certain sulfites, including those that are commonly added to wine as preservatives. However, since there are several winemakers who add sulfites to none of the wines they produce, people who would like to drink wine but are allergic to sulfites can drink wines produced by these winemakers without risking an allergic reaction to sulfites. Which of the following is an assumption on which the argument depends? (A) These winemakers have been able to duplicate the preservative effect produced by adding sulfites by means that do not involve adding any potentially allergenic substances to their wine. (B) Not all forms of sulfite are equally likely to produce the allergic reaction. (C) Wine is the only beverage to which sulfites are commonly added. (D) Apart from sulfites, there are no substances commonly present in wine that give rise to an allergic reaction. (E) Sulfites are not naturally present in the wines produced by these winemakers in amounts large enough to produce an allergic reaction in someone who drinks these wines. 177. In the United States, of the people who moved from one state to another when they retired, the percentage who retired to Florida has decreased by three percentage points over the past ten years. Since many local businesses in Florida cater to retirees, these declines are likely to have a noticeably negative economic effect on these businesses and therefore on the economy of Florida. Which of the following, if true, most seriously weakens the argument given? (A) People who moved from one state to another when they retired moved a greater distance, on average, last year than such people did ten years ago. (B) People were more likely to retire to North Carolina from another state last year than people were ten years ago. (C) The number of people who moved from one state to another when they retired has increased significantly over the past ten years.

(D) The number of people who left Florida when they retired to live in another state was greater last year than it was ten years ago. (E) Florida attracts more people who move from one state to another when they retire than does any other state. 178. Businesses are suffering because of a lack of money available for development loans. To help businesses, the government plans to modify the income-tax structure in order to induce individual taxpayers to put a larger portion of their incomes into retirement savings accounts, because as more money is deposited in such accounts, more money becomes available to borrowers. Which of the following, if true, raises the most serious doubt regarding the effectiveness of the governments plan to increase the amount of money available for development loans for businesses? (A) When levels of personal retirement savings increase, consumer borrowing always increases correspondingly. (B) The increased tax revenue the government would receive as a result of business expansion would not offset the loss in revenue from personal income taxes during the first year of the plan. (C) Even with tax incentives, some people will choose not to increase their levels of retirement savings. (D) Bankers generally will not continue to lend money to businesses whose prospective earnings are insufficient to meet their loan repayment schedules. (E) The modified tax structure would give all taxpayers, regardless of their incomes, the same tax savings for a given increase in their retirement savings. 179. As of 2011, eight seasons have been broadcast and the show has been renewed for one more season. Each episode is titled with a quotation from it; unlike most episodic television, the episode title usually bears no relationship to any of the plotlines; the quote fragment being completely out of context. The show's 100th episode ("City of Great Racks") aired on October 15, 2007. To celebrate this, a casino-inspired party was held at West Hollywood's Pacific Design Center. Warner Brothers Television also distributed blue Micargi Rover bicycles adorned with the Two and a Half Men logo along with the words "100 Episodes." Each bicycle came with a note saying "You've made us very proud. Here's to a long ride together." The cast also gifted the crew with sterling silver key rings from Tiffany & Co. The key rings were attached to small pendants with "100" inscribed on one side and Two and a Half Men on the other. Which of the following can't be concluded from the above passage? 1)Two and a Half Men is a very successful show. 2)The person naming every episode of the show has a quirky sense of humour. 3)Having a title that has no relation to the content of the episode is standard practice in episodic television. 4)They gave away blue Micargo Rover bicycles to celebrate the 100th episode of Two and a Half Men. 5)Warner Bros. television had something to do with the show. 180. Life expectancy in the UK will continue to rise, along with the rest of Europe, despite fears over the impact of obesity, population experts have said. Analysing trends from the past 40 years, the experts credited a decline in deaths from heart disease for

the continued rise. Deaths from cardiovascular disease in the UK have seen, some of the largest and most rapid falls of any Western European country, partly due to improvements in treatment as well as reductions in smoking and other risk factors. The latest figures from 2009 show that life expectancy in the UK is 82.6 years for women and 78.4 years for men. Which of the following, if true, would most seriously challenge the experts view on life expectancy in the UK? 1. People in the US are living much longer than those in the UK. 2. Gross domestic product and health care expenditure per capita are generally good predictors of population health within high-income countries. 3. The impact on mortality of the current generation of people who have been obese from childhood through adulthood is not available yet. 4. There is widespread incidence of type-2 diabetes among teenagers due to obesity. 5. The impact of widespread occurrence of type-2 diabetes among teenagers due to obesity has not been linked to mortality rates. 1) 2) 3) 4) 5) ii and iii i, ii and iv iii only ii and v iii and iv

181. In a recent study, professor of psychiatry Harrison Pope put dissociative amnesia i.e., 'repressed memory' - to the test of time. He reasoned that if dissociative amnesia were an innate capability of the human brain - akin to depression, hallucinations, anxiety and dementia - it would appear in written works throughout history. What is Professor Pope's basic assumption? 1) Written works throughout history would accurately record representative samples of the psychological abnormalities of human beings. 2) The human brain has a multitude of capabilities, including depression, hallucinations, etc. 3) Historical records may provide a clue regarding the causes and cure of dissociative amnesia. 4) Dissociative amnesia has so far been assumed to be a recent psychological phenomenon. 5) Dissociative amnesia is in fact not an innate capability of the human brain. 182. London, the city often denigrated as 'the Smoke', is the dirtiest EU capital apart from Athens, according to a survey. Severe traffic congestion causing high levels of air pollution and problems with rubbish collection combine to make the city an environmental black spot. These are sprawling cities with public transport problems and severe traffic congestion, producing a detrimental effect on air quality, said Slagin Parakatil, senior researcher at William M. Mercer, the consulting firm which produced the survey. Waste disposal systems are also under pressure from the cities' dense populations, making them less efficient than in other EU cities.'

Which of the following could seriously undermine the argument put forth by Mr. Parakatil, as regards London? 1) New evidence has come to light which suggests that pollution levels in London have remained consistently high due to the presence of textile industries on the outskirts of the city. 2) Pollution levels in both London and Athens have been the lowest in the past month as compared to the last six months. 3) Nearly 30% of the owners of two- as well as four-wheelers in London have installed pollution control technology into their vehicles in the last 6 months. 4) Pollution levels in London are high not because of traffic problems and inefficient waste disposal systems, but because of the increasing presence of polluting industrial units within the city itself. 5) The credibility of the method and apparatus used to measure pollution levels in London is being challenged in the courts of Britain. 183. In Los Angeles, a political candidate who buys saturation radio advertising will get maximum name recognition. The statement above logically conveys which of the following? A.Radio advertising is the most important factor in political campaigns in Los Angeles. B.Maximum name recognition in Los Angeles will help a candidate to win a higher percentage of votes cast in the city. C.Saturation radio advertising reaches every demographically distinct sector of the voting population of Los Angeles. D.For maximum name recognition a candidate need not spend on media channels other than radio advertising. E.A candidate's record of achievement in the Los Angeles area will do little to affect his or her name recognition there. 184. The rate of violent crime in this state is up 30 percent from last year. The fault lies entirely in our court system: Recently our judges' sentences have been so lenient that criminals can now do almost anything without fear of a long prison term. The argument above would be weakened if it were true that A.85 percent of the other states in the nation have lower crime rates than does this state. B.white collar crime in this state has also increased by over 25 percent in the last year. C.35 percent of the police in this state have been laid off in the last year due to budget cuts. D.polls show that 65 percent of the population in this state oppose capital punishment. E.the state has hired 25 new judges in the last year to compensate for deaths and retirements. 185. The increase in the number of newspaper articles exposed as fabrications serves to bolster the contention that publishers are more interested in boosting circulation than in printing the truth. Even minor publications have staffs to check such obvious fraud. The argument above assumes that A.newspaper stories exposed as fabrications are a recent phenomenon. B.everything a newspaper prints must be factually verifiable. C.fact checking is more comprehensive for minor publications than for major ones.

D.only recently have newspapers admitted to publishing intentionally fraudulent stories. E.the publishers of newspapers are the people who decide what to print in their newspapers. 186. Time and again it has been shown that students who attend colleges with low faculty/student ratios get the most well-rounded education. As a result, when my children are ready to attend college, I'll be sure they attend a school with a very small student population. Which of the following, if true, identifies the greatest flaw in the reasoning above? A.A low faculty/student ratio is the effect of a well-rounded education, not its source. B.Intelligence should be considered the result of childhood environment, not advanced education. C.A very small student population does not by itself, ensure a low faculty/student ratio. D.Parental desires and preferences rarely determines a child's choice of a college or university. E.Students must take advantage of the low faculty/student ratio by intentionally choosing small classes. 187. All German philosphers, except for Marx, are idealists. From which of the following can the statement above be most properly inferred? A.Except for Marx, if someone is an idealist philosopher, then he or she is German. B.Marx is the only non-German philosopher who is an idealist. C.If a German is an idealist, then he or she is a philosopher, as long as he or she is not Marx. D.Marx is not an idealist German philosopher. E.Aside from the philosopher Marx, if someone is a German, then he or she is an idealist. 188. Reviewing the historical data researchers in California found that countries with the largest number of pieces of health and fitness equipment per head have had the lowest incidence of death due to cardiac problems. The researchers have concluded that people in these countries exercise regularly and therefore do not suffer from cardiac problems The researchers conclusion could would be more strengthened if which of the following were true:1.The more health equipments one has the more time one spends in using them. 2.The incidence of cardiac problems in countries with the largest number of health and fitness equipments is likely to decrease further. 3. Occupation and lifestyles affect the risk of health problem less than use of health and fitness equipments. 4.The more health and fitness equipments people have the greater is their awareness of the need for exercise to keep the cardiac problems away. 189. Companies considering new cost-cutting manufacturing processes often compare the projected results of making the investment against the alternative of not making the investment with costs, selling prices, and share of market remaining constant. Which of the following, assuming that each is a realistic possibility, constitutes the most serious disadvantage for companies of using the method above for evaluating the financial benefit of new manufacturing processes? a. The costs of materials required by the new process might not be known with certainty. b. In several years interest rates might go down, reducing the interest costs of

borrowing money to pay for the investment. c. Some cost-cutting processes might require such expensive investments that there would be no net gain for many years, until the investment was paid for by savings in the manufacturing process. d. Competitors that do invest in a new process might reduce their selling prices and thus take market share away from companies that do not. e. The period of year chosen for averaging out the cost of the investment might be somewhat longer or shorter, thus affecting the result. 190. While Governor Verdant has been in office, the state's budget has increased by an average of 6 percent each year. While the previous governor was in office, the state's budget increased by an average of 11.5 percent each year. Obviously, the austere budgets during Governor Verdant's term have caused the slowdown in the growth in state spending. Which of the following, if true, would most seriously weaken the conclusion drawn above? a. The rate of inflation in the state averaged 10 percent each year during the previous governor's term in office and 3 percent each year during Verdant's term. b. Both federal and state income tax rates have been lowered considerably during Verdant's term in office. c. In each year of Verdant's term in office, the state's budget has shown some increase in spending over the previous year. d. During Verdant's term in office, the state has either discontinued or begun to charge private citizens for numerous services that the state offered free to citizens during the previous governor's term. e. During the previous governor's term in office, the state introduced several so-called "austerity" budgets intended to reduce the growth in state spending. 191. The people who are least likely to be audited by the Internal Revenue Service this year are those who have been audited since 1985 and who were found to have made no mistakes in filing their returns during that audit. Of the following people, who is MOST likely to be audited by the IRS? (A) A person who was audited in 1986 but was not found to have made any mistakes in filing his return. (B) A person who was audited in 1986 and whose lawyer corrected several mistakes in the tax return prior to the filing deadline. (C) A person whose spouse was convicted of tax fraud in 1987, who was then audited and found to have made no mistakes. (D) A person who was last audited in 1984, and had no mistakes uncovered by the IRS during that audit. (E) A person who was audited in each of the past five years, but was found to have made no mistakes in any of the filings 192. Linsey has been judged to be a bad songwriter simply because her lyrics typically are disjointed and subjective. This judgment is ill founded, however, since the writings of many modern novelists typically are disjointed and subjective and yet these novelists are widely held to be good writers.

Which one of the following is an assumption on which the argument depends? (A) Disjointed and subjective writing has a comparable effect in modern novels and in songs. (B) Some readers do not appreciate the subtleties of the disjointed and subjective style adopted by modern novelists. (C) Song lyrics that are disjointed and subjective have at least as much narrative structure as any other song lyrics do. (D) A disjointed and subjective style of writing is usually more suitable for novels and song lyrics than it is for any other written works. (E) The quality of Linseys songs is better judged by quality of their lyrics than by the quality of their musical form. 193. Recent studies have highlighted the harmful effects of additives in food (colors, preservatives, flavor enhancers etc.). There are no synthetic substances in the foods we produce at Munchon Foods we use only natural ingredients. Hence you can be sure you are safeguarding your familys health when you buy our products. Which of the following, if true, would most weaken the contention of Munchon Foods? A. Some synthetic substances are not harmful B. Some natural substances found in foods can be harmful C. Food without additives is unlikely to taste good D. Munchon Foods produces only breakfast cereals E. Without preservatives some foods could cause harm 194. A fruit known as amla in certain parts of Asia is an excellent source of vitamin C. A small quantity of the fruit grated and added to salads provides almost all the daily requirement of this vitamin. However, the fruit is very sour. A new process designed to remove most of the sour taste will make the fruit acceptable to American tastes. We are therefore starting to grow this fruit for sale in the United States. The argument above assumes all of the following except A. Americans generally won't eat very sour foods B. The new process does not remove a significant part of the vitamin content C. That a market exists for a new source of vitamin C D. The fruit can be used only in salads E. Apart from being sour there are no other objections to eating this fruit 195. Anton: I sold my house on an internet site last year and was happy with the price. I got a speedy sale and the cost of advertising was insignificant. I would advise you to avoid real estate agents. Barbie: It is in the interest of the real estate agent to get me the best price for my property because he gets a commission based on the selling price. Therefore, when selling my house I will certainly use an agent rather than trying to sell the house by word of mouth, or by advertising in newspapers or on the internet. Barbie's could strengthen her position by pointing out all of the following except A. Houses of comparable value often obtain a lower price when sold on the internet B. Very few houses are sold on the internet at the moment an so a valid comparison is difficult C. The agent's service includes many add-on benefits in terms of legal fees, surveyor's reports and advice that are not available on internet sites

D. Some buyers pay the agent to find them a cheap house E. The agent's commission is usually less than the difference between the internet price and the higher price the agent obtains for you 196. Early data on seat-belt use showed that seat-belt wearers were less likely to be killed in road accidents. Hence, it was initially believed that wearing a seat-belt increased survival chances in an accident. But what the early analysts had failed to see was that cautious drivers were more likely to wear the belts and were also less likely to cause 'big accidents', while reckless drivers were more likely to be involved in 'big' accidents and were less likely to wear the belts. Which of the following, if true, could an opponent of the view presented above best cite as a reason for recommending continued use of seat-belts? A. Careful drivers who are involved in accidents caused by reckless drivers, would be more likely to survive if wearing a belt B. All drivers should be required by law to wear a belt C. The ratio of 'big' to 'small' road accidents is very small D. In fatal accidents seat-belt wearers in the front seat are less likely to survive than those wearing seat belts in the back seat E. On average, careful drivers pay lower insurance premiums than do drivers who have been involved in accidents. 197. On the basis of the Big Bang theory scientists predicted levels of Helium-3 in the universe that are ten times greater than the levels actually observed. According to the original model, Helium-3 is produced when low-mass stars burn up hydrogen and become red giants, as well as being produced in the Big Bang itself. Researchers have now produced a new model in which the Helium-3 produced by a red giant is pushed to the stars interior and burnt up. Hence the Big Bang theory is no longer undermined by Helium-3 data. The two portions in bold-face are related to each other in which of the following ways? A. The first highlights an observation that tends to undermine a particular theory. The second is that theory. B. The first is a fact that undermines a theory. The second is context for accepting that theory. C. The first points to an inconsistency in a particular model; the second is the authors main conclusion. D. The first is a challenge to a classic theory; the second resolves that challenge. E. The first is a position that the author does not accept; the second is the authors position. 198. Sue: Commercial flights currently contribute more carbon dioxide to the atmosphere in one year than does the whole of Africa. If we want to reduce global warming we need to restrict the number of flights we take. Dave: Did you know that by taking one inter-continental flight you cause more pollution than you would in twelve months of car travel? Daves response to Sues comment serves to A. reinforce Sues contention that flights are a major contributor to increased carbon

dioxide levels B. add more weight to her contention that we should reduce the number of flights we take C. mitigate the force of her argument by suggesting that there is an alternative approach D. suggest an alternative that will reduce the effect of pollution E. question whether she really understands the severity of global warming 199. It is not unusual to see the ball fall into a black slot on a roulette wheel four times in a row. But for it to fall five or six times in a row into the same color is very unusual. Therefore you can win money by waiting for a run of five of the same color and then betting against that color. If the roulette wheel in question is a fair wheel, which of the following observations or facts, if it were true, would best reveal a fallacy in the logic? A. If there were a reliable way to win at roulette it would be well-known by now. B. It is hard for a player to keep track of what went before for the time required. C. The probability of getting a particular color decreases with the number of times the color has appeared. D. The probability of getting a particular color is always the same no matter what has gone before. E. A person who makes money this way once or twice, will carry on to lose that money after a few more times. 200. 1 Josh has twenty years of typing experience behind him; therefore, if you are looking for an efficient typist to enter your data into the new system, you need look no further. The speaker assumes that A. Twenty years of practice ensures typing efficiency B. The type of typing required for the new system is identical to what Josh has been doing C. Josh's job profile is the best that the new employer is going to get D. Josh is an outstandingly fast and accurate typist E. Josh will fit well into the new office 201. It has been suggested that long-term prisoners, on release from jail, be given a reasonable state pension to reduce the likelihood of their resorting to crime. Most people instinctively reject the suggestion as they feel it would be like rewarding criminal activity. The supporters of the prisoners' pension scheme have criticized those who reject this possibility, by claiming that for the critics...... Which of the following is the most logical completion of the sentence above? A. emotion is more important than justice B. punishment for criminals is more important than crime prevention C. crime prevention is not an important issue D. money has too high a value E. the law should not be concerned with what happens after jail

202. The rate of violent crime in this state is up 30 percent from last year. The fault lies entirely in our court system: Recently our judges' sentences have been so lenient that criminals can now do almost anything without fear of a long prison term. The argument above would be weakened if it were true that A. 85 percent of the other states in the nation have lower crime rates than does this state. B. white collar crime in this state has also increased by over 25 percent in the last year. C. 35 percent of the police in this state have been laid off in the last year due to budget cuts. D. polls show that 65 percent of the population in this state oppose capital punishment. E. the state has hired 25 new judges in the last year to compensate for deaths and retirements. 203. The increase in the number of newspaper articles exposed as fabrications serves to bolster the contention that publishers are more interested in boosting circulation than in printing the truth. Even minor publications have staffs to check such obvious fraud. The argument above assumes that A. newspaper stories exposed as fabrications are a recent phenomenon. B. everything a newspaper prints must be factually verifiable. C. fact checking is more comprehensive for minor publications than for major ones. D. only recently have newspapers admitted to publishing intentionally fraudulent stories. E. the publishers of newspapers are the people who decide what to print in their newspapers. 204. Time and again it has been shown that students who attend colleges with low faculty/student ratios get the most well-rounded education. As a result, when my children are ready to attend college, I'll be sure they attend a school with a very small student population. Which of the following, if true, identifies the greatest flaw in the reasoning above? A. A low faculty/student ratio is the effect of a well-rounded education, not its source. B. Intelligence should be considered the result of childhood environment, not advanced education. C. A very small student population does not by itself, ensure a low faculty/student ratio. D. Parental desires and preferences rarely determine a child's choice of a college or university. E. Students must take advantage of the low faculty/student ratio by intentionally choosing small classes. 205. Bill earns more commission than does Sandra. But since Andrew earns more commission than does Lisa, it follows that Bill earns more commission than does Lisa. Any of the following, if introduced into the argument as an additional premise, makes the argument above logically correct EXCEPT: A.Andrew earns more commission than Bill B.Sandra earns more commission than Lisa

C.Sandra earns more commission than Andrew D.Sandra and Andrew earn the same amount of commission E.Bill and Andrew earn the same amount of commission 206. During the SARS days, about 23,500 doctors who had treated SARS sufferers died and about 23,670 doctors who had not engaged in treatment for SARS sufferers died. On the basis of those figures, it can be concluded that it was not much more dangerous to participate in SARS treatment during the SARS day than it was not to participate in SARS treatment. Which of the following would reveal most clearly the absurdity of the conclusion drawn above? A. Counting deaths among doctors who had participated in SARS treatment in addition to deaths among doctors who had not participated in SARS treatment B. Expressing the difference between the numbers of deaths among doctors who had treated SARS sufferers and doctors who had not treated SARS suffers as a percentage of the total number of deaths C. Separating deaths caused by accidents during the treatment to SARS suffers from deaths caused by infect of SARS suffers. D. Comparing death rates per thousand members of each group rather than comparing total numbers of deaths E. Comparing deaths caused by accidents in the United States to deaths caused by infect in treating SARS suffers. 207. In 2003 an airline in United State lost more than half, on average, of the foreign passengers they had previously served each year. Researchers have alleged that this extreme drop resulted from a rise in price of tickets for international lines from $60 to $90 per 1,000 miles. Which of the following, if feasible, offers the best prospects for alleviating the problem of the drop in passengers as the researchers assessed it? Cooperating with other airlines to provide more international lines. Allowing foreign passengers to pay the same as the previous international line Reemphasizing the goals and mission of the airline as serving both domestic passengers and foreign passengers Increasing the financial resources of the airline by raising the ticket price for domestic passengers Offering superior VIP service for foreign passengers. 208. A life insurance company allows people to prepay their endowment insurance at current rates. The policyholder then pays the premium every year. People should participate in the program as a means of decreasing the cost for their living after retirement. Which of the following, if true, is the most appropriate reason for people NOT to participate in the program? a.Peoples are unsure about which insurance company they will choose after retirement. b.The amount of money accumulated by putting the prepayment funds in an interestbearing account today will be greater than the total cost of insurance when they retire. c.The annual cost of premium is expected to increase at a faster rate than the annual increase in the cost of living. d.Some of the insurance companies are contemplating large increases in premium next year. e.The prepayment plan would not cover the cost of hospitalization.

209. If highways were restricted to cars and only those truck with capacity of less than 8 tons, most the truck traffic would be forced to run outside highway. Such a reduction in the amount of truck traffic would reduce the risk of collision in highway. i)The conclusion draw in the first sentence depends on which of the following assumptions? The roads outside highway would be as convenient as highway for most drivers of truck. Most roads outside highways are not ready to handle truck traffic. Most trucks that are currently running in highway have a capacity of more than 8 tons. Cars are at greater risk of becoming involved in collisions than are trucks. A reduction in the risk of collision would eventually lead to increases in car traffic. ii)Which of the following, if true, would most strengthen the conclusion drawn in the second sentence? A.Cars with a capacity of more than 8 tons are already excluded outside highways. B.Highways are experiencing overcrowded traffic primarily because of sharp increases in car traffic. C. Many drivers of trucks would rather buy truck with a capacity of less than 8 tons than be excluded from highways. D. The number of collisions that occur near highways has decreased in recent years. F.Trucks that have a capacity of more than 8 tons cause a disproportionately large number of collisions in highways. 210. Any truthful auto mechanic will tell you that your standard 5000 miles check up can detect only one fifth of the problems , which are likely to go wrong with your car. Therefore such a check up is virtually worthless and a waste of time and money. Which of the following , if true, weakens the above conclusion. 1)Those problems which the 5000 miles check up will turn up with are the ten leading causes of the engine failure. 2)For a new car. a 5000 miles check up is required to protect the owner's warranty. 3)During the 5000 miles check up , the mechanic also performs the routine maintenance, which is necessary for the proper functioning of the car. a)1 only b)2 only c) 1,2,3 d)2 & 3 211. The spinning jenny is a multi-spool spinning wheel It was invented in 1764 by James Hargreaves. The device dramatically reduced the amount of work needed to produce yarn, with a single worker able to work eight or more spools at once. In 1778, Samuel Crompton invented the spinning mule combining the spinning jenny with Richard Arkwrights spinning frame and again dramatically increasing yarn production. The spinning jenny was so effective in increasing the efforts of a workers labour that Karl Marx cited it as one of the causes behind the elimination of slavery.Which of the following statements, if true, would most significantly strengthen the conclusion drawn by Karl Marx in the passage (1) Before the invention of the spinning jenny the number of slaves employed in the cotton plantations for harvesting raw cotton had been on the rise (2) Before the invention of the jenny more slaves were employed in cotton plantations for producing yarn than for harvesting raw cotton (3) After the invention of the jenny the number of slaves employed in producing yarn in cotton plantations fell dramatically compared to the number employed in other jobs on such plantations

(4) The invention of the spinning jenny made it possible for workers using it to charge more for their skills (5) The regular use of the spinning jenny marked the beginning of the Industrial Revolution in many countries

212.Our work has proven to be very successful in the past three years; each of our five clients has experienced the fastest growth of sales in their history. Therefore, if your company wants to increase sales, do not hesitate to call Singh and Mathur since we are the solution. Which of the following, if true, weakens the argument presented above? (1) Most of the consultants at Singh and Mathur hold MBA degrees. (2) Even without the help of Singh and Mathur, the five clients of Singh and Mathur would have achieved the same growth rate in sales. (3) Singh and Mathur do not provide a complete range of services. (4) Singh and Mathur use an updated accounting approach to help companies cut cost. (5) Singh and Mathur have a dynamic DBMS. 213. Most scientists agree that new lines of interdisciplinary research are the need of the hour. Even government committees on science have stressed the need for more interdisciplinary projects. Yet, of ten proposals for new interdisciplinary projects last year, only one was successfully funded. Some have suggested that this means that as yet researchers are not coming up with sufficiently persuasive projects, or that their proposals are not of high enough quality, or even that the reputations of these researchers is not high enough. However, the real reason probably lies in the way funding is organized. Funding is still allocated according to the old categories and there are no funds specifically for research that overlaps different subject areas. The two parts in bold-face are related to each other in which of the following ways? A. The first is a finding that the author finds unacceptable; the second is the authors own position B. The first is a finding that the author attempts to account for; the second is a finding that contradicts the authors main conclusion. C. The first is a fact that the author attempts to account for. The second is data that explicitly supports the authors main conclusion. D. The first is a position that the author opposes; the second is the authors main position. E. The first is a situation that the author finds paradoxical; the second is an assumption that the author uses to reinforce the paradox. 214. One of the possible results of the further depletion of the ozone in the atmosphere would be a sharp increase in the demand for sunscreens. Many of the ingredients in sunscreens are natural, and the demand for these products will require strict conservation laws. This will represent the victory for those who desire greater protection for the environment. Which one of the following best states the conclusion in the passage above? A. Industries that produce ozone-depleting chemicals should be encouraged to continue doing so. B. Regulation of ozone-depleting chemicals should be handled on a federal level. C. The natural ingredients in sunscreen products should be replaced by synthetic substitutes. D. The effects of ozone depletion on the environment are not categorically negative. E. The few positive effects of ozone depletion are far outweighed by the myriad negative

effects. 215. The American division of the Acme running shoe company should begin marketing running shoes with double-thick soles. Acmes Swedish division is selling a large number of these items, and is enjoying its most profitable quarter ever. Which of the following is an assumption made by the author of the argument above? A. Acmes Brazilian division is also selling a large number of running shoes with doublethick soles. B. Swedes and Americans have similar tastes in music. C. Acme is the only running shoe company that manufacturers double-thick soles. D. The running shoe market in America tends to follow the same trends as does the market in Sweden. E. Acme has experienced financial troubles in the last three quarters. 216. Currently about 70 percent of all urban households recycle some portion of their garbage, 25 percent more than did ten years ago. This increase can be attributed , in part , to the fact that a larger portion of the population is now under the age of 47. Of the following, which can be most properly inferred from the passage above? A. Of those who currently recycle, at least 70 percent recycle less than 25 percent of what is potentially recyclable. B. Younger people are generally more conscientious about waste disposal than are older people. C. People under the age of 47 are more likely to recycle than are their older counterparts. D. The overall population has risen by at least 25 percent in the past ten years. E. People currently over the age of 47 recycled more often ten years ago. 217. The price of purchasing a car in Country Q is 120 percent less than the price of purchasing a car in Country Y. Even after transportation fees and tariff charges are added, it is still cheaper for a buyer to import car from Country Q to Country Y than to buy car in Country Y. The statements above, if true, best support which of the following assertions?

Gasoline prices in Country Q are 120 percent below those in Country Y. Importing cars from Country Q to Country Y will eliminate 120 percent of the sales of cars in Country Y. The tariff on a car imported from Country Q to Country Y is less than 120 percent of the price of a car in Country Y. The fee for transporting a car from Country Q to Country Y is more than 120 percent of the price of a car in Country Q. It takes 120 percent less time to transport a car in Country Q than it does in Country Y.

218..In 1992, 5 percent of every dollar paid in tax went to support the unemployed citizens. In 1998, 8 percent of every dollar paid in tax went to such funds, although that unemployment rate has decreased in 1998 than in 1992. Each of the following, if true, could explain the simultaneous increase in percent of every dollar paid in tax to support the unemployed citizens and decrease in the number of

unemployment rate EXCEPT: On average, each unemployed citizen received more money in 1998 than 1992. On average, people paid less tax in 1998 than in 1992. The individuals had paid more tax than did enterprises during this period. Income before tax has significantly decreased since 1992. The number of tax evaders rose sharply between 1992 and 1998.

219.Something must be done to stop spam. In early days, people seldom received unsolicited email advertisement; but now that numerous bulk email software and email address finders are developed to collect email address all around the world. Advertisers use email addresses to market their products and even sell such email lists to other advertisers. As a result, almost everyone ever get junk email, and sometime several and even tens of annoying emails a day. So, relevant anti-spam regulations should be framed to stop unsolicited advertising. The two portions in boldface play which of the following roles? a.Background that the argument depends on and conclusion that can be drawn from the argument. b. Part of evidence that the argument includes, and inference that can be drawn from this passage. c. Pre-evidence that the argument depends on and part of evidence that supports the conclusion. d.Background that argument depends on and part of evidence that supports the conclusion. e.Pre-evidence that argument includes and a method that helps to supports that conclusion. 220. The society which, by fixing itself in place locally, gives space a content by arranging individualised places, thus finds itself enclosed inside this localisation. This is equivalent to: 1] Racing, with its preoccupation on speed is a sport that survives in spite of close encounters with death. 2] Women's magazines, with emphasis on beauty, sexuality and independence are actually fixated on capitalism since they are by-products of consumerism. 3] Psychiatrists maintain that reading habits when begun early create a whole world of imagination in a child's mind that is of great developmental importance. 4] The paraphernalia of the armed forces are merely a semiotic of aggression and antagonistic war dances 221. To understand the origins of the word 'classic', we need to return to the ancient Greeks, who by peculiar good fortune and natural enlightenment of mind, had no classics but themselves. They were at first the only classical authors for the Romans, coming just after, who strove and contrived to imitate them. Which of the following if true would strengthen the argument of this passage? 1] At first the only true classics were the ancient Greeks.

2] The Greeks were in a class by themselves. 3] The Greeks were revered by the Romans who had a love of Greek art. 4] The Greeks and Romans together laid foundations for classical art. 222. Self-consciousness exists in itself and for itself, in that, and by the fact that it exists for another self-consciousness; that is to say, it 'is' only by being acknowledged or "recognised." This statement is equivalent to: 1] Tobacco companies spend increasingly on point-of-purchase merchandise since the consumer is most vulnerable at the moment of choice. 2] Offices increasingly have reflective glass in their lobbies since anti-social behaviour is reduced when people are continually aware of how they appear. 3] Video cameras are installed in most shopping malls in order for the store to monitor the goings-on not just among customers but also among staff. 4] To become a hit playback singer it is necessary to be recognised as the number one in a particular genre of music. 223. A true classic, as I should like to hear it defined, is an author who has enriched the human mind, who has spoken to all in his own peculiar style, a style which is found to be also that of the whole world, a style new without neologism, new and old, easily contemporary with all time. We can infer that: 1] A classic, according to the usual definition, is an old author canonised by admiration, and an authority in his particular style. 2] A classic may for a moment have been revolutionary; it may at least have seemed so, but it is simply a subversion of whatever prevented the restoration of the balance of order and beauty. 3] A classic refers to those who have become models in any language whatever, have established the expressions, models, fixed rules for composition and style, strict rules of art to which men must conform. 4] A classical author is one who has discovered some moral and not equivocal truth, or revealed some eternal passion or desire. 224. In the historical society divided into classes, culture is the general sphere of knowledge and of representations of the lived; which is to say that culture is the power of generalisation existing apart, as division of intellectual labour and as intellectual labour of division. From the passage above, culture is clearly: 1] A means of segregating the classes 2] A natural product of a historical society 3] A factor of the intellect 4] A generalised means of historicising society

225. If a bottle is to be selected at random from a certain collection of bottles, what is the probability that the bottle will be defective? (1) The ratio of the number of bottles in the collection that are defective to the number that are not defective is 3:500. (2) The collection contains 3,521 bottles. Options: A) Both statements 1 and 2 together are sufficient to answer the question but neither statement is sufficient alone. B) Statement 1 alone is sufficient but statement 2 alone is not sufficient to answer the question asked. C) Each statement alone is sufficient to answer the question. D) Statement 2 alone is sufficient but statement 1 alone is not sufficient to answer the question asked. E) Statements 1 and 2 are not sufficient to answer the question asked and additional data is needed to answer the statements. 226. Mr. Janeck: I dont believe Stevenson will win the election for governor. Few voters are willing to elect a businessman with no political experience to such a responsible public office. Ms. Siuzdak: Youre wrong. The experience of running a major corporation is a valuable preparation for the task of running a state government. M. Siuzdaks response shows that she has interpreted Mr. Janecks remark to imply which of the following? (A) Mr. Janeck considers Stevenson unqualified for the office of governor. (B) No candidate without political experience has ever been elected governor of a state. (C) Mr. Janeck believes that political leadership and business leadership are closely analogous. (D) A career spent in the pursuit of profit can be an impediment to ones ability to run a state government fairly. (E) Voters generally overestimate the value of political experience when selecting a candidate. 227. The writer can only be fertile if he renews himself and he can only renew himself, if his soul is constantly enriched by fresh experience. Which of the following is most likely to support the above thought? (1) Only out of fresh experience can the writer get germs for new writing. (2) The writer can meet new people. (3) The writer must see new places. (4) None of these 228. But because the idea of private property has been permitted to override with its selfishness, the common good of humanity it does not follow that there are not limits within which that idea can function for the general convenience and advantage. Which of the following is most likely to weaken the argument? (1) All the people of the society should progress at an equitable rate and there should be no disparities and private property does bring about a tremendous disparity. (2) One should not strive for the common good of humanity at all, instead one should be concerned with maximising ones own wealth. (3) One should learn from the experiences of former communist nations and should not repeat his mistakes at all.

(4) Even prosperous capitalist countries like the USA have their share of social problems. 229. I have been studying it, consciously and subconsciously, for 40 years and I still find men unaccountable; people I know intimately can surprise me by some action of theirs which I never thought them capable of or by the discovery of some trait which exhibit a side of themselves that I never even suspected. The idea in this sentence can be best summarised as (1) men are inconsistent and therefore one should not be confident even about ones closest friends. (2) men are unpredictable, one can never tell what they will do next; hence, one should be very careful in ones dealings. (3) no matter how closely you know somebody there still exists an unknown facet of his personality. (4) None of these 230. Three airlines IA, JA and SA operate on the Delhi- Mumbai route. To increase the number of seats sold, SA reduced its fares and this was emulated by IA and JA immediately. The general belief was that the volume of air travel between Delhi and Mumbai would increase as a result. Which of the following, if true, would add credence to the general belief? (1) Increase in profitability of the three airlines. (2) Extension of the discount scheme to other routes. (3) A study that shows that air travellers in India are price-conscious. (4) A study that shows that as much as 80 per cent of air travel in India is company sponsored. 231. When there is less rainfall than normal, the water level of Australian rivers falls and the rivers fl ow more slowly. Because algae whose habitat is river water grow best in slow-moving water, the amount of algae per unit of water generally increases when there has been little rain. By contrast, however, following a period of extreme drought, algae levels are low even in very slow-moving river water. Which of the following, if true, does most to explain the contrast described above? (A) During periods of extreme drought, the populations of some of the species that feed on algae tend to fall. (B) The more slowly water moves, the more conducive its temperature is to the growth of algae. (C) When algae populations reach very high levels, conditions within the river can become toxic for some of the other species that normally live there. (D) Australian rivers dry up completely for short intervals in periods of extreme drought. (E) Except during periods of extreme drought, algae levels tend to be higher in rivers in which the fl ow has been controlled by damming than in rivers that flow freely. 232. When hypnotized subjects are told that they are deaf and are then asked whether they can hear the hypnotist, they reply, "No." Some theorists try to explain this result by arguing that the

selves of hypnotized subjects are dissociated into separate parts, and that the part that is deaf is dissociated from the part that replies. Which of the following challenges indicates the most serious weakness in the attempted explanation described above? (A) Why does the part that replies not answer, "Yes"? (B) Why are the observed facts in need of any special explanation? (C) Why do the subjects appear to accept the hypnotist's suggestion that they are deaf? (D) Why do hypnotized subjects all respond the same way in the situation described? (E) Why are the separate parts of the self the same for all subjects? 233. A prominent investor who holds a large stake in the Burton Tool Company has recently claimed that the company is mismanaged, citing as evidence the company's failure to slow production in response to a recent rise in its inventory of finished products. It is doubtful whether an investor's sniping at management can ever be anything other than counterproductive, but in this case it is clearly not justified. It is true that an increased inventory of finished products often indicates that production is outstripping demand, butin Burton's case it indicates no such thing. Rather, the increase in inventory is entirely attributable to products that have already been assigned to orders received from customers. In the argument given, the two boldfaced portions play which of the following roles? (A) The fi rst states the position that the argument as a whole opposes; the second provides evidence to undermine the support for the position being opposed. (B) The fi rst states the position that the argument as a whole opposes; the second is evidence that has been used to support the position being opposed. (C) The fi rst states the position that the argument as a whole opposes; the second states the conclusion of the argument as a whole. (D) The fi rst is evidence that has been used to support a position that the argument as a whole opposes; the second provides information to undermine the force of that evidence. (E) The first is evidence that has been used to support a position that the argument as a whole opposes; the second states the conclusion of the argument as a whole. 235. Products sold under a brand name used to command premium prices because, in general, they were superior to nonbrand rival products. Technical expertise in product development has become so widespread, however, that special quality advantages are very hard to obtain these days and even harder to maintain. As a consequence, brandname products generally neither offer higher quality nor sell at higher prices. Paradoxically, brand names are a bigger marketing advantage than ever. Which of the following, if true, most helps to resolve the paradox outlined above? (A) Brand names are taken by consumers as a guarantee of getting a product as good as the best rival products. (B) Consumers recognize that the quality of products sold under invariant brand names can drift over time.

(C) In many acquisitions of one corporation by another, the acquiring corporation is interested more in acquiring the right to use certain brand names than in acquiring existing production facilities. (D) In the days when special quality advantages were easier to obtain than they are now, it was also easier to get new brand names established. (E) The advertising of a companys brand-name products is at times transferred to a new advertising agency, especially when sales are declining 236. Hotco oil burners, designed to be used in asphalt plants, are so efficient that Hotco will sell one to the Clifton Asphalt plant for no payment other than the cost savings between the total amount the asphalt plant actually paid for oil using its former burner during the last two years and the total amount it will pay for oil using the Hotco burner during the next two years. On installation, the plant will make an estimated payment, which will be adjusted after two years to equal the actual cost savings. Which of the following, if it occurred, would constitute a disadvantage for Hotco of the plan described above? (A) Another manufacturers introduction to the market of a similarly efficient burner (B) The Clifton Asphalt plants need for more than one new burner (C) Very poor efficiency in the Clifton Asphalt plants old burner (D) A decrease in the demand for asphalt (E) A steady increase in the price of oil beginning soon after the new burner is installed 237. Poly-chlorinated biphenyls, a versatile group of chemicals used in Michigan in recent years to aid in the cultivation of animal feed grain, should be banned immediately lest they cause more harm to humans. Residual traces of the biphenyls used in the cultivation of feed grain have been found in the cow feed distributed to hundreds of Michigan farmers over the past three years. Which of the following most conclusively strengthens the argument above? (A) Scientists have observed an unusually high rate of intestinal cancer among consumers of Michigan dairy products over the past two years. (B) Traces of poly-chlorinated biphenyls have been found in the blood and urine of many Michigan cows. (C) Agricultural scientists have raised doubts as to the effectiveness of poly-chlorinated biphenyls in promoting the growth of nutritious grain products for cows. (D) Industrial scientists who contributed to the research and development of polychlorinated biphenyls conducted extensive tests to ascertain the safety of the chemicals. (E) The rate of sterility among Michigan cows greatly exceeds the national average. 238. We doubt that the latest government report will scare Americans away from ham, bacon, sausages, hot dogs, bologna, and salami or that it will empty out the bars or cause a run on natural food supplies. If a diet were to be mandated from Washington, Americans probably would order the exact opposite course. Therefore, the diet that does make sense is to eat a balanced and varied diet composed of foods from all food groups and containing a reasonable caloric intake. Which of the following is (are) specifically implied by the passage? I. Vitamins are necessary to combat disease.

II. A recent report warned of the risks of meat and alcoholic beverages. III. Unorthodox suggestions for a more nutritional diet were recently made by the government. A. I only B. II only C. III only D. I and II only E. II and III only 239.The new vehicle inspection program is needed to protect the quality of the states air, for us and for our children. Auto exhausts are a leading contributor to coughing, wheezing, choking, and pollution. The states long-term interests in the health of its citizens and in this area as a place to live, work, and conduct business depend on clean air. Which of the following is an unstated assumption made by the author? A. Working and conducting business may be different activities. B. The Minister believes that Indian companies are looking for business expansion.The state has been interested in the health of its citizens even before this inspection program was proposed. C. Exhaust emissions contribute to pollution. D. The new inspection program will be effective. E. Our ancestors did not suffer from air pollution 240. Which of the following is the most logical completion of the passage below? In the 1940s, the introduction of the 33 rpm long-playing vinyl record completely changed the way we listen to music. The breakable and three minute 78 rpm record soon disappeared from the marketplace. In our day, the compact disk, superior in quality and convenience, has replaced the vinyl long-playing record and will . . . A. increase the size of the record-buying public. B. increase the profits of the record industry. C. drive the 78 rpm record from the second-hand market. D. make the manufacture of phonographs that play 33 rpm records unnecessary. E. encourage the growth of computer-generated music 241.In most economies, the government plays a role in the market system. Governments enforce the rules of the game, impose taxes, and may control prices through price ceilings or price supports. These actions necessarily may create shortages or surpluses. In most developed and interdependent economies, the necessity of the governments playing some role in the economy is disputed. The author of the passage would probably agree that A. economic surpluses are always good. B. market shortages are a necessary evil. C. higher prices strengthen the economy. D. price ceilings add to the shortages. E. surpluses are not usually created intentionally 242.That the policy of nuclear deterrence has worked thus far is unquestionable. Since

the end of the Second World War, the very fact that there were nuclear armaments in existence has kept major powers from using nuclear weapons, for fear of starting a worldwide nuclear exchange that would make the land of the power initiating it uninhabitable. The proof is that a third world war between superpowers has not happened. Which one of the following, if true, indicates a flaw in the argument? A. Maintaining a high level of nuclear armaments represents a significant drain on a countrys economy. B. From what has happened in the past, it is impossible to infer with certainty what will happen in the future, so an accident could still trigger a third world war between superpowers. C. Continuing to produce nuclear weapons beyond the minimum needed for deterrence increases the likelihood of a nuclear accident. D. The major powers have engaged in many smaller-scale military operations since the end of the Second World War, while refraining from a nuclear confrontation. E. It cannot be known whether it was nuclear deterrence that worked, or some other factor, such as recognition of the economic value of remaining at peace 243. One of the truisms of the advertising industry is that it is rarely necessary to say something of substance in an advertisement in order to boost sales. Instead, one only needs to attract the potential customers attention; memory does the rest, for it is more important for sales that people know of a product than that they know something about it. Which of the following is assumed by the argument? A. People can remember a product without having much information about it. B. Advertisements, in their own way, function to improve peoples memories. C. Attracting a potential customers attention is a simple matter. D. The advertising industry knows little of substance about the products it promotes. E. Advertisements seldom tell the truth about a product 244.Why save endangered species? For the general public, endangered species appear to be little more than biological oddities. A very different perception is gained from considering the issue of extinction in a wider context. The important point is that many major social advances have been made on the basis of life forms whose worth would never have been perceived in advance. Consider the impact of rubber-producing plants on contemporary life and industry: approximately two-thirds of the worlds rubber supply comes from rubber producing plants and is made into objects as diverse as rubber washers and rubber boots. The point of the passage is made chiefly by A. acknowledging the validity of two opposing points of view B. appealing to the emotions of the audience rather than to their intellects C. suggesting a useful perspective for viewing the question raised at the beginning of the passage D. trying to discredit the view of an opponent without presenting an alternative hypothesis 245. In a recent report, the gross enrolment ratios at the primary level, that is, the number of children enrolled in classes one to five as a proportion of all children aged 6 to 10, were shown to be very high for most states; in many cases they were way above 100 percent! These figures are not worth anything, since they are based on the official enrolment data compiled from school records. They might as well stand for gross exaggeration ratios.

Which of the following options best supports the claim that the ratios are exaggerated? The definition of gross enrolment ratio does not exclude, in its numerator, children below 6 years or above 10 years enrolled in classes one to five. A school attendance study found that many children enrolled in the school records were not meeting a minimum attendance requirement of 80 percent. A study estimated that close to 22 percent of children enrolled in the class one records were below 6 years of age and still to start going to school. Demographic surveys show shifts in the population profile which indicate that the number of children in the age group 6 to 10 years is declining. 246. Szymanski suggests that the problem of racism in football may be present even today. He begins by verifying an earlier hypothesis that clubs wage bills explain 90% of their performance. Thus, if players salaries were to be only based on their abilities, clubs that spend more should finish higher. If there is pay discrimination against some group of players -- fewer teams bidding for black players thus lowering the salaries for blacks with the same ability as whites -- that neat relation may no longer hold. He concludes that certain clubs seem to have achieved much less than what they could have, by not recruiting black players. Which of the following findings would best support Szymanskis conclusions? Certain clubs took advantage of the situation by hiring above-average shares of black players. Clubs hired white players at relatively high wages and did not show proportionately good performance. During the study period, clubs in towns with a history of discrimination against blacks, under-performed relative to their wage bills. Clubs in one region, which had higher proportions of black players, had significantly lower wage bills than their counterparts in another region which had predominantly white players. 247. United Lumber will use trees from its forest for two products. The tree trunks will be used for lumber and the branches converted into wood chips to make fiberboard. The cost of this conversion would be the same whether done at logging site, where the trees are debranched, or at the United's factory. However, wood chips occupy less than half the volume of the branches from which they are made. The information given, if accurate, most strongly supports which of the following? A. Converting the branches into wood chips at the logging site would require transporting a fully assembled wood-chipping machine to and from the site. B. It would be more economical to debranch the trees at the factory where the fireboard is manufactured. C. The debranching of trees and the conversion of the branches into chips are the the only stages in the processing of branches that would be in the economic advantage to perform at the logging site. D. Transportation costs from the logging site to the factory that are determined by volume of the cargo would be lower if the conversion into chips is done at the logging site rather than at the factory. E. In the wood- processing industry, branches are used only for the production of wood chips for fibre board. 248. In response to mounting public concern, an airplane manufacturer implemented a program with the well-publicized goal of reducing by half the total yearly amount of hazardous waste generated by its passenger-jet division. When the program began in

1994, the division's hazardous waste output was 90 pounds per production worker; last year it was 40 pounds per production worker. Clearly, therefore, charges that the manufacturer's program has not met its goal are false. Which of the following is an assumption on which the argument depends? A. The amount of nonhazardous waste generated each year by the passenger-jet division has not increased significantly since 1994. B. At least as many passenger jets were produced by the division last year as had been produced in 1994. C. Since 1994, other divisions in the company have achieved reductions in hazardous waste output that are at least equal to that achieved in the passenger-jet division. D. The average number of weekly hours per production worker in the passenger-jet division was not significantly greater last year than it was in 1994. E. The number of production workers assigned to the passenger-jet division was not significantly less in 1994 than it was last year 249.Consumers are not so easily manipulated as they are often painted. They may know what they want, and what they want may be greatly different from what other people believe they need. Which of the following statements, if true, most weakens the above argument? a) Most people continue to buy the same brand of a product year after year. b) Companies that advertise the most sell the most products c) Store shelves packed with a variety of different brands have the potential to confuse the consumer. d) most consumers know which brand they are going to buy before entering the store 250. Official sources put the number of child labourers in the country at 17 million of which two million are engaged in hazardous occupations. On the other hand, while some independent sources estimate the number to be closer to 44 million, there are others who have fixed the figure at 100 million. Which of the following could be the fundamental reason for the conflicting reports on the size of child labour? a) Accuracy of official sources. b) Inaccuracy of independent sources. c) Lack of agreement between official figures and figures of independent sources in all areas. d) prevailing confusion on the question of what constitutes child labour

Vous aimerez peut-être aussi